Добавил:
Upload Опубликованный материал нарушает ваши авторские права? Сообщите нам.
Вуз: Предмет: Файл:
База (модуль1)-патофізіологія.doc
Скачиваний:
228
Добавлен:
18.02.2016
Размер:
735.23 Кб
Скачать

ПАТОЛОГІЧНА ФІЗІОЛОГІЯ

Нозологія

1. У хворого з ознаками недостатності мітрального клапана в анамнезі відмічалась атака ревматизму, що супроводжувалася запальними явищами в суглобах. Яке з патологічних явищ у даного хворого відноситься до категорії «патологічного стану»?

A.*Недостатність мітрального клапана

B. Ревматизм

C. Артрит

D. Запалення суглобів

E. Ревмокардит

2. У хворого відзначається атрофія альвеолярних відростків щелепи після видалення зубів. Це є прикладом:

A.*Патологічного стану

B. Патологічної реакції

C. Патологічного процесу

D. Структурного сліду адаптації

E. Хвороби

3. У жінки, віком 55 років, стоматолог виявив кілька зубів уражених карієсом. Під яке визначення підлягає дане явище?

A. *Патологічний процес.

B. Патологічний стан.

C. Патологічна реакція

D. Хвороба.

E. Компенсаторна реакція

4. У хворого з обширними опіками шкіри тулуба є виражені ознаки інтоксикації. Для якої стадії опікової хвороби це характерно?

A. *Опікової токсемії

B. Опікового шоку

C. Опікової інфекції

D. Опікового виснаження

E. Термінальної

5. При рентгенологічному обстеженні хворого з виразковою хворобою шлунка виявлено стеноз пілоруса. Це порушення є:

A. *Патологічним станом

B. Патологічним процесом

C. Хворобою

D. Патологічною реакцією

E.

6. Чоловік, 30 років, був опромінений дозою близько 3-ох Грей. Які зміни в крові будуть через 8 годин після опромінення?

A. *Лімфопенія.

B. Лейкопенія.

C. Гранулоцитопенія.

D. Тромбоцитопенія.

E. Анемія.

7. Чоловік потрапив до лікарні через 3 доби після впливу іонізуючого випромінювання в дозі 3 Грея. Зміни з боку якої фізіологічної системи в першу чергу слід очікувати у нього?

A. *Крові

B. Серцево-судинної

C. Імунної

D. Травлення

E. Ендокринної

8. У хворого діагностовано кістково-мозкову форму гострої променевої хвороби. В крові виявлено лейкоцитоз та лімфопенію. Для якої стадії хвороби характерні ці зміни?

A. *Первинних проявів

B. Прихованої

C. Розпалу хвороби

D. Завершення хвороби

E. Віддалених наслідків

9. У популяції населення України частота гетерозигот за геном фенілкетонурії дорівнює 3%. Який метод дослідження використовується в генетиці для раннього виявлення фенілкетонурії у новонароджених?

A. *Біохімічний

B. Популяційно-статистичний

C. Генеалогічний

D. Цитогенетичний

E. Дерматогліфіки

10. У чоловіка, 52 років, ясна різців верхньої щелепи темно-червоного кольору, гноєтеча з ясеневих закутків, розхитування зубів. Діагностований парадонтит. Який період хвороби у цього чоловіка?

A. *Виражених проявів.

B. Передхвороба.

C. Латентний.

D. Продромальний.

E. Кінець хвороби.

11. Наприкінці зими студент, який останнім часом відчував психічне перевантаження, захворів на гостре респіраторне захворювання після переохолодження. Що є причиною цього?

A. *Патогенний збудник.

B. Психічне перевантаження.

C. Переохолодження.

D. Нераціональне харчування.

E. Гіповітаміноз.

12. Ліквідатор аварії на ЧАЕС звернувся до лікаря зі скаргами на виражену слабість, крововиливи на шкірі, проноси. Аналіз крові: ШОЕ-25 мм/год, еритроцити 2,4х1012/л, лейкоцити 2,2х109/л, тромбоцити – 70х109/л. Для якої фази гострої променевої хвороби характерна дана клінічна картина?

A. *Розпалу хвороби

B. Первинної гострої реакції

C. Вдаваного клінічного благополуччя

D. Відновлення

E. Кінця хвороби

13. Серед багатьох факторів, які визначають тяжкість електротравми, першочергове значення має ураження внутрішніх органів, розташованих на шляху проходження струму. Ураження якого з них є найнебезпечнішим?

A.*Серця

B. Наднирників

C. Легень

D. Печінки

E. Нирок

14. У результаті пошкодження одного з реакторів АЕС стався витік радіоактивних продуктів. Люди, які знаходилися в зоні підвищеної радіації, орієнтовно отримали по 250-300 Р. Їх негайно госпіталізовано. Який ведучий симптом буде характерний у потерпілих ?

A. *Лімфопенія

B. Лейкопенія

C. Анемія

D. Тромбоцитопенія

E. Нейтропенія

15. При роботі по ліквідації наслідків аварії на АЕС робітник отримав дозу опромінення 500 рентген. Скаржиться на головний біль, нудоту, часті запаморочення. Які зміни лейкоцитів можна спостерігати у хворого через 10 годин після опромінення?

A. * Нейтрофільний лейкоцитоз

B. Лімфоцитоз

C. Лейкопенія

D. Агранулоцитоз

E. Еозинофілія

16. При екстреній евакуації з підводного човна, що знаходився на глибині 100 м, у матросів виникли судоми, втрата свідомості. Який основний патогенетичний механізм при цих порушеннях?

А.*Газова емболія.

B.Токсична дія азоту.

C.Токсична дія СО2.

D.Токсична дія кисню.

E.Гіпоксія.

17. В експерименті на щурах викликали електротравму, періодично пропускаючи побутовий електричний струм через організм тварин, внаслідок чого виникли певні пошкодження органів і систем організму. Які з них характерні для проявів загального адаптаційного синдрому (ЗАС)?

A.*Геморагічні виразки слизової шлунку і 12-ти палої кишки

B.Фібриляція серцевого мяза

C.Артеріальна гіпотензія

D.Тонічні судоми

E.Зупинка дихання

18. В лікарню до кінця робочого дня доставлений робітник “гарячого” цеху, який скаржиться на головний біль, головокружіння, нудоту, загальну слабкість. Свідомість збережена, шкірні покриви гіперемовані, сухі, гарячі на дотик. ЧСС – 130/хв.. Дихання часте, поверхневе. Яке порушення процесів регуляції тепла найвірогідніше виникло в чоловіка в даній ситуації?

A.*Зниження тепловіддачі

B.Посилення тепловіддачі і зниження теплопродукції

C.Посилення тепловіддачі і теплопродукції

D.Посилення теплопродукції без зміни тепловіддачі

E.Зниження теплопродукції без зміни тепловіддачі

19. Хворий, вперше поступив в стаціонар з діагнозом – виразкова хвороба шлунка. На даний час скаржиться на болі в епігастральній ділянці, печію, нудоту, дьогтеподібний стілець. Як можна охарактеризувати такий стан хворого?

A.*Ускладнення

B. Рецидив

C. Ремісія

D. Патологічна реакція

E. Патологічний стан

20. Працівникові атомної станції, який отримав радіоактивне опромінення, був введений радіопротектор. Який з перерахованих механізмів можна віднести до основних патофізіологічних механізмів радіопротекції?

A.* Зниження утворення вільних радикалів

B . Запобігання тканинної гіпоксії

C . Підвищення інтенсивності окислювальних реакцій

D. Підвищення кровопостачання тканин

E. Створення у клітині надлишку НАДФ Н2

21.У хворого, з опіком рук, який потрапив в лікарню в стані алкогольного сп’яніння, через добу розвинувся абстинентний синдром. Який патогенетичний механізм зумовлює виникнення абстинентного синдрому при алкоголізмі?

A.* Зниження синтезу ендогенного алкоголю

B. Розвиток сенсибілізації нервових клітин до алкоголю

C. Формування патологічної толерантності клітин організму

D. Виникнення поліорганної функціональної недостатності

E. Зниження активності алкогольдегідрогенази в печінці

22. Тривале перебування людини в космосі викликало деструктивні зміни в кістках. Яка основна причина цього явища?

A.*Від’ємний Са++ баланс

B. Гіподинамія

C. Незначні навантаження на рухову систему

D. Психологічні порушення

E. Невагомість

23. Відомо, що типові патологічні процеси розвиваються за однаковими закономірностями в різних органах і тканинах та у різних видів тварин. Назвіть з перерахованих типовий патологічний процес:

A.*Пухлина

B. Туберкульоз

C. Гіпертонічна хвороба

D. Непрохідність кишечника

E. Інфаркт міокарда

24. При розвитку обмороження у людини спостерігається поблідніння шкіри відкритих ділянок тіла, знижується її температура. Який механізм лежить в основі цих змін ?

A .* Відкриття артеріовенозних анастомозів

B . Розширення судин шкіри і підшкірної клітковини

C . Звуження судин внутрішніх органів

D. Зменшення числа серцевих скорочень

E. Закриття артеріовенозних анастомозів

25. При зануренні на глибину 75 метрів у водолаза через деякий час почала спостерігатися втрата чіткого мислення. Дією якої речовини це зумовлено?

A. *Азоту

B. Кисню

C. Вуглекислого газу

D. Гелію

E. Оксигемоглобіну

26. У туристів в горах на висоті 3 500 метрів виникли : слабкість, головний біль, шум у вухах, часте дихання, серцебиття, тобто перші ознаки гірської хвороби. Який ведучий механізм лежить в основі її виникнення?

A.*Гіпоксемія

B . Алкалоз

C. Гіперкапнія

D. Гіпокапнія

E. Гіпобарія

27. Пацієнт проходить у клініці курс лікування з приводу гостроі променевої хвороби, яка супроводжується розвитком гіпопластичної анемії. Кілька днів тому відмічалася кровотеча з шлунково- кишкового тракту. Який найбільш ймовірний механізм виникнення геморагічного синдрому в даному випадку?

A.*Зниження продукції тромбоцитів

B. Зниження продукції еритроцитів

C. Зниження проліферації лейкоцитів

D. Прискорене руйнування еритроцитів у судинному руслі

E. Внутріклітинний гемоліз еритроцитів

28. Ураження хворого одноразовою дозою іонізуючого випромінювання спричинило розвиток кістково-мозкової форми ГПХ. Які патологічні прояви будуть характерними в період удаваного благополуччя?

A. * Наростаюча лімфопенія, лейкопенія

B. Перерозподільний лейкоцитоз, лімфоцитоз

C. Анемія, лейкопенія

D. Тромбоцитопенія, анемія

E. Тромбоцитопенія, лейкоцитоз

29. У хворого з виразковою хворобою після проведеного лікування нормалізувалось харчове травлення, зникли болі, покращився настрій. Але через кілька тижнів знову з’явились болі в епігастрїі, печія, відрижка кислим. Як ви охарактеризуєте таке протікання хвороби?

A. *Рецидив хвороби

B. Період ремісії

C. Ускладнення хвороби

D. Продромальный період

E. Латентний період

30. У хворого з наркотичною залежністю внаслідок відсутності наступної дози наркотику развинувся абстинентний синдром. Який патогенетичний механізм обумовив иникнення абстинентного синдрому при наркоманії?

A.*Зниження синтезу ендогенних опіатів

B. Зниження активності ферментів, метаболізуючих наркотик

C. Розвиток сенсибілізації нервових клітин до алкоголю

D. Формування патологічної толерантності клітин організму

E. -

31. Морфологічні та біохімічні зміни, які відбуваються у старіючому організмі, торкаються в першу чергу клітин. Який механізм ушкодження клітин при цьому є провідним?

A.*Зниження енергетичного забезпечення

B. Зміни в будові ДНК і РНК

C. Зниження мембранного потенціалу клітин

D. Посилення перекисного окислення ліпідів

E. Підвищення проникності мембран лізосом

32. Студент ІІІ курсу на вулиці біля інституту купив пиріжки з м’ясом і з’їв їх. Через 2 години відчув біль у шлунку, нудоту, головний біль, мерехтіння перед очима. Згодом виникли блювота і пронос. Що це було?

A *Патологічний процес

B Патологічна реакція

C Патологічний стан

D Рецидив гастриту

E Алергічна реакція

Спадкова патологія. Конституція

1. У|в,біля| молодої людини| чоловічої статі віком 20 років високого| зросту|зріст| та астенічної будови| тіла з ознаками| гіпогонадизму, гінекомастією та зменшеною| продукцією сперми| [азооспермія] виявлено| каріотип 47 ХХY. Який спадковий| синдром супроводжується такою хромосомною аномалією?

A. *Клайнфельтера

B. Віскотта-Олдрича

C. Тернера

D. Луї-Барра

E. Дауна

2. При обстеженні юнака| з розумовою| відсталістю виявлено| євнухоїдну будову| тіла, недорозвиненість статевих| органів. У клітинах порожнини| рота - статевий| хроматин|. Який метод генетичного| дослідження слід застосувати| для уточнення| діагнозу?

A. *Цитологічний

B. Біохімічний

C. Клініко-генеалогічний

D. Популяційно-статистичний

E. Дерматогліфіки

3. У медико-генетичну| консультацію за рекомендацією андролога| звернувся| чоловік 35 років з приводу відхилень фізичного і психічного розвитку|. Об'єктивно встановлено|: високий| зріст, астенічна будова| тіла, гінекомастія, розумова| відсталість. При мікроскопії клітин слизової оболонки| ротової порожнини| знайдено| в 30\% статевий| хроматин| (одне| тільце Барра). Який найбільш вірогідний діагноз?

A.* Синдром Клайнфельтера.

B. Синдром Ді Джорджі.

C. Хвороба Дауна

D. Хвороба Реклінгаузена.

E. Хвороба Іценка-Кушінга.

4. При диспансерному обстеженні хлопчикові 7 років виставлено| діагноз - дальтонізм. Батьки здорові, кольоровий| зір нормальний|. Але у|в,біля| дідуся по материнській лінії така| ж аномалія. Який тип успадкування| цієї аномалії?

A.* Рецесивний, зчеплений| зі статтю|

B. Домінантний, зчеплений| зі статтю|

C. Неповне домінування

D. Аутосомно-рецесивний

E. Аутосомно-домінантний

5. Хворий 12 років поступив|вчинивши| в клініку з гемартрозом колінного суглобу|. З анамнезу відомо, що з раннього| дитинства| страждає кровоточивістю. Яке захворювання у|в,біля| хлопчика?

A. *Гемофілія.

B. Геморрагічний васкуліт.

C. Гемолітична анемія.

D. Вітамін В12 (фолієво) -дефіцитна анемія.

E. Тромбоцитопенічна пурпура.

6. У|в,біля| чоловіка| 32 років високий| зріст, гінекомастія, оволосіння за жіночим типом, високий тембр| голосу, розумова| відсталість, безпліддя. Попередній діагноз – синдром Клайнфельтера. Для його| уточнення| необхідно дослідити:

A. *Каріотип

B. Лейкоцитарну формулу

C. Сперматогенез

D. Групу крові

E. Родовід

7. Чоловік 70 років страждає на подагричний| артрит. У його| родоводі також| були| хворі на подагру. Який чинник|фактор| є безпосередньою| причиною розвитку| патології в данному випадку?|

A. *Генетичний дефект обміну сечової кислоти|

B. Генетичний дефект обміну сечовини|

C. Похилій вік

D. Надмірне споживання| м’яса|

E. Чоловіча стать

8. У|в,біля| молодих| здорових| батьків народилася дівчинка, білява, з голубими| очима|.У перші ж місяці життя| у|в,біля| дитини| розвинулись| дратівливість, неспокій, порушення| сну і харчування|, а обстеження| невропатолога показало відставання в розвитку||. Який метод генетичного| обстеження| дитини| слід застосувати| для уточнення| діагнозу ?

A. *Біохімічний

B. Цитологічний

C. Близнюковий

D. Генеалогічний

E. Популяційно-статистичний

9. Хвора 9 р. протягом першого| року|рок| життя| знаходилась| на природному вигодовуванні. В кінці першого| року|рок| перенесла пневмонію|в, із затяжним| перебігом. Ходити почала пізно. З’явились телеангіоектазії на шкірі і кон’юнктиві|. У крові – відсутність Ig A, знижений| рівень Т-лімфоцитів. Яке з імунодефіцитних захворювань| у|в,біля| дівчинки?

A. *Синдром Луї-Барр

B. Синдром Віскотта-Олдрича

C. Синдром Чедіака-Хігасі

D. Імунодефіцит швейцарського| типу

E. Синдром Ді Джорджі

10. У|в,біля| хлопчика 5 міс., при дослідженні імунного статусу виявлено| зменшення| імуноглобулінів, особливо IgA та IgM. В крові та лімфатичних вузлах| відсутні В-лімфоцити та плазматичні клітини. Реакції Т-лімфоцитів збережені. Захворювання передається по спадковості як зчеплене| зі статтю|. Який діагноз буде| найбільш ймовірним?

A. * Хвороба Брутона

B. Синдром Луї-Барр

C. Синдром Віскотта-Олдрича

D. Імунодефіцит швейцарського| типу

E. Рання гіпогаммаглобулінемія

11. До лікарні поступив|вчинивши| юнак| 17 років зі скаргами| на затримку| росту|зріст|, посивіння, облисіння, зниження| гостроти| зору| і слуху|чутка|, гіперкератоз. В крові гіперглікемія, гіперхолестеринемія. Для якоі патології характерні ці явища|?

A. * Синдром Вернера.

B. Синдром Гетчинсона-Гілфорда.

C. Синдром Шерешевського-Тернера.

D. Сімейна гіперхолестеринемія.

E. Синдром Клайнфельтера.

12. У медико-генетичній консультації при обстеженні хворого хлопчика в крові були| виявлені нейтрофільні лейкоцити| з 1 “барабанною паличкою|”. Наявність якого| синдрому можлива| в дитини?|в,біля|

A. * Синдром Клайнфельтера.

B. Синдром Дауна

C. Синдром Шерешевського-Тернера

D. Синдром Едвардса

E. Синдром трисомії – Х

13. У медико-генетичну| консультацію звернулася| жінка по рекомендації гінеколога з приводу відхилень фізичного і статевого| розвитку|. При мікроскопії клітин слизової ротової порожнини| не знайдено| статевого| хроматину|. Який | найбільш вірогідний діагноз в даному випадку?

A.* Синдром Шерешевського-Тернера

B. Хвороба Дауна

C. Синдром Клайнфельтера

D. Хвороба Реклінгаузена

E. Трисомія по Х-хромосомі

14. При обстеженні дитини| з олігофренією виявлено| в крові і спинномозковій рідині підвищений вміст фенілаланіну і фенілпіровиноградної кислоти|. Реакція сечі з трихлороцтовим| залізом позитивна. Вкажіть з порушенням| синтезу якого| ферменту пов'язаний| розвиток| захворювання|?

A. *Фенілаланінгідроксилаза.

B. Оксидаза п-гідроксифенілпіровиноградної кислоти|.

C. Тирозиназа.

D. Дофамінгідроксилаза.

E. Оксидаза гомогентизинової кислоти|.

15. Який метод генетичного| обстеження| найбільш достовірно встановить| діагноз синдрому Шерешевського - Тернера у|в,біля| хворої людини|?

A.* Виявлення статевого| хроматину|.

B. Підрахунок "барабанних| паличок|".

C. Генеалогічний.

D. Близнюковий.

E. Дерматогліфіки.

16. Чоловік 26 років скаржиться| на безпліддя. Об'єктивно: зріст 186 см, довгі кінцівки, гінекомастія, гіпоплазія яєчок, в зішкрібі слизової оболонки| щоки| знайдені тільця Барра. Діагностований синдром Клайнфельтера. Який механізм хромосомної аномалії має місце в даному випадку?|

A. *Нерозходження| хромосом| у|в,біля| мейозі

B. Нерозходження| хроматид| у|в,біля| мітозі

C. Транслокація

D. Інверсія хромосоми|

E. Делеція хромосоми|

17. У|в,біля| дитини| з геморагічним синдромом діагностована гемофілія В. Вона зумовлена| дефіцитом фактора:|фактор|

A. *ІХ (Крістмаса)

B. ІІ (протромбіну)

C. VІІІ (антигемофільного глобуліну)

D. ХІ (протромбопластину|)

E. ХІІ (Хагемана)

18. У|в,біля| хлопчика 15 років, хворого на алкаптонурію сеча набуває чорного| кольору| після відстоювання. Спадкове порушення| обміну якої речовини| має місце:

A. *Тирозину

B. Цистеїну

C. Аланіну

D. Сечовини

E. Сечової кислоти|

19. У|в,біля| 40-річної вагітної проведено амніоцентез. При дослідженні каріотипу плоду отримано|отримано| результат: 47,ХУ+21. Яку патологію плоду виявлено|?

A.* Синдром Дауна

B. Синдром Клайнфельтера

C. Хвороба Шерешевського-Тернера

D. Фенілкетонурія

E. Хвороба Патау

20. При обстеженні букального| епітелію чоловіка було виявлено статевий хроматин|. Для якої хромосомної аномалії це характерно|вдача|?

A.* Синдром Клайнфельтера

B. Хвороба Дауна

C. Синдром Шерешевського-Тернера

D. Трисомія по Х-хромосомі

E. Гіпофосфатемічний рахіт

21. Виникнення нижчеперерахованих| захворювань| пов’язане| з генетичними| чинниками|фактор|.Назвіть патологію зі спадковою| схильністю.

A. *Цукровий діабет

B. Хорея Гентінгтона

C. Фенілкетонурія

D. Серповидноклітинна анемія

E. Дальтонізм

22. У немовляти відмічається блювота, діарея, загальна|спільний| дистрофія, гепато-| і спленомегалія. При припиненні годування молоком – симптоми зменшуються. Відсутність якого ферменту лежить в основі спадкового|спадкоємний| порушення обміну речовин?

A. *Галактози|

B. Триптофану|

C. Тирозину|

D. Фенілаланіну|

E. Глюкозо-6-фосфатдегідрогенази|

23. У дитини,яка часто хворіє ангінами та фарингітами, відмічається збільшення лімфовузлів і селезінки. Зовнішній вигляд характеризується пастозністю і блідістю,м’язова тканина розвинута слабо. В крові спостерігається лімфоцитоз. Як називається такий вид діатезу?

A.*Лімфатико-гіпопластичний

B. Ексудативно-катаральний

C. Нервово-артритичний

D. Астенічний

E. Геморагічний

24. При обстеженні 12-ти літнього хлопчика, який відстає в рості, знайдена ахондроплазія: непропорційна тілобудова з помітним вкороченням рук і ніг внаслідок порушення росту епіфізарних хрящів довгих трубчастих

кісток. Дана хвороба є:

A. *Спадковим за домінантним типом

B. Спадковим за рецесивним типом

C. Спадковим зчепленим зі статтю

D. Вродженим

E. Набутим

25. При проведенні амніоцентезу в клітинах плода виявлено по 2 тільця статевого хроматину (тільця Барра). Для якого захворювання характерна дана ознака?

A. * Трисомія X

B. Синдром Клайнфельтера

C. Синдром Шерешевського-Тернера

D. Синдром Дауна

E. Синдром Патау

26. У пацієнта виявлено брахідактилію. За яким типом успадковується дана патологія?

A. *Аутосомно - домінантний

B. Аутосомно – рецесивний

C. Неповне домінування

D. Зчеплений з Х – хромосомою домінантний

E. Зчеплений з Х – хромосомою рецесивний

27. У подружжя народилася дитина з хворобою Дауна. Матері 42 роки. Назвіть вид порушень внутрішньоутробного розвитку, який найбільш імовірно призвів до даної хвороби.

A. *Гаметопатія

B. Бластопатія

C. Ембріопатія

D. Неспецифічна фетопатія

E. Специфічна фетопатія

28. У дитини виявлена схильність до ожиріння, яка є результатом діатезу. Назвіть вид діатезу, при якому частіше може розвинутись ожиріння.

A. *Нервово-артритичний

B. Ексудативно-катаральний

C. Лімфатико-гіпопластичний

D. Астенічний

E -

29. При алкаптонурії відбувається надмірне виділення гомогентизинової кислоти із сечею. З порушенням метаболізму якої амінокислоти пов’язано виникнення цього захворювання?

A.*Тирозину

B. Фенілаланіну

C. Аланіну

D. Метіоніну

E. Аспарагіну

30. Відомо, що дія мутагенів, а також генетичних факторів служить розвитку спадкових хвороб, центральною ланкою патогенезу яких є мутації. Який фактор викликає мутацію ?

A. *Віруси

B. Атмосферний тиск

C. Дія низької температури

D. Електричний струм

E. Перегрівання

31. У хлопчика 12 років встановлено інфаркт міокарду, у крові значно підвищений рівень ліпопротеїнів ( ЛП ) низької щільності. Поставлено діагноз - сімейна гіперхолестеринемія. Рецептори для яких речовин відсутні на клітинних мембранах при цьому спадковому захворюванні?

A.*ЛП низької щільності

B. ЛП високої щільності

C. Інсуліну

D. Тироксину

E. Глюкагону

32. Відомо, що ймовірність виникнення атеросклерозу є значно вищою у сім’ях, де вже хтось хворів на цю недугу. Це пов’язано з тим, що атеросклероз є:

A. *Хвороба зі спадковою схильністю

B. Вродженою хворобою

C. Хромосомною хворобою

D. Молекулярно-генетичною хворобою з домінантним типом успадкування

E. Молекулярно-генетичною хворобою з неповним домінантним успадкуванням

33. Новонароджена дитина відмовляється від їжі, в неї періодичне блювання, пронос, а з часом виникло помутніння кришталика. Про яку патологію слід думати лікарю?

A. *Галактоземія

B. Тирозиноз

C. Фенілкетонурія

D. Цистинурія

E. Алкаптонурія

35. Жінка С., 35 років потрапила в автомобільну аварію, отримала струс мозку. Невдовзі після виписки з лікарні з’явились перші розлади у психіці, що поглиблювались і через рік їй був поставлений діагноз – шизофренія. Дослідження родоводу показало, що серед двоюрідних і троюрідних сибсів також є шизофреніки. У даному випадку шизофренія є:

A. *Хворобою зі спадковою схильністю

B. Спадковою хворобою

C. Вродженою хворобою

D. Набутою хворобою

E. Віддаленим наслідком травми

36. Хвороба Вільсона–Коновалова – спадкове захворювання, яке характеризується гепатоцеребральною дистрофією.Успадкування за аутосомно-рецесивним типом. Для виявлення захворювання мають значення екзогенні фактори, вражаючі печінку (інтоксикації, інфекції). Основну роль в патогенезі гепатоцеребральної дистрофії мають генетично обумовлені порушення обміну

A. *Білків і міді

B. Белків і кальцію

C. Ліпідів і кальцію

D. Вуглеводів і кальцію

E. Вуглеводів і калію

37. Фруктоземія – це спадкове захворювання, обумовлене різким зниженням активності ферменту фруктозо – 1 – фосфатальдолази. Це захворювання зустрічаєтьсязс частотою 1:20000 населення. За яким типом успадковується фруктоземія?

A.* Аутосомно-рецесивне спадкування

B. Аутосомно-домінантне спадкування

C. Х –зчеплене- домінантне спадкування

D. Х –зчеплене –рецесивне спадкування

E.

38. В абсолютній більшості випадків хронічного мієлолейкозу у клітинах пухлинного клону є "філадельфійська" хромосома. Яка подія відбувається із стовбуровою кровотворною клітиною кісткового мозку внаслідок зазначеної хромосомної аберації?

A.*Трансформація

B. Прогресія

C. Припинення поділу

D. Порушення дозрівання

E. Вихід у кров

39. Хлопчик 7 років хворіє на гіпогаммаглобулінемію Брутона. Який тип успадкування характерний для цього захворювання?

A. *Рецесивний, зчеплений з Х-хромосомою

B. Домінантний, зчеплений з Х-хромосомою

C. Аутосомно-рецесивний

D. Аутосомно-домінантний

E. Неповне домінування

40. Відомо, що є зв’язок між морфологічними особливостями людини та схильністю до певних психічних захворювань. Хто запропонував класифікацію, що відображає цей зв’язок?

A.*Кречмер

B. Гіппократ

C. Сіго

D. Чорноруцький

E. Павлов

41. У дитини 3 років пастозний вигляд, часті запалення з вираженим ексудатом, схильністю до тривалого перебігу та алергічних реакцій. Для якого діатезу характерні ці явища ?

A.* Ексудативного

B. Астенічного

C. Лімфатико-гіпопластичного

D. Нервово-артритичного

E. Геморагічного

42. У дитини п'яти років, що часто хворіє на респіраторні захворювання, відмічаються екзематозні явища після прийому деяких харчових продуктів, схильність до затяжного перебігу запальних процесів. Який вид діатезу можна припустити передбачити в даному випадку?

A.* Ексудативно-катаральний

B. Геморрагічний

C. Нервово-артритичний

D. Лімфатико-гіпопластичний

E. Астенічний

43. У дитини тиміко - лімфатичний статус. Однією з його ознак є:

A. *Збільшення загрудинної залози

B. Зменшення загрудинної залози

C. Збільшення щитоподібної залози

D. Зменшення щитоподібної залози

E. Збільшення паращитоподібних залоз

44.У новонародженого хлопчика спостерігається деформація мозкового та лицьового черепа, мікрофтальмія, деформація вушної раковини, вовча паща, і т.ін. Каріотип дитини - 47,XY,13+. Про яку хворобу йде мова?

A. *Синдром Патау

B. Синдром Едвардса

C. Синдром Дауна

D. Синдром Клайнфельтера

E. Синдром Шерешевського-Тернера

46. До лікарні доставлено дитину 2-х років з уповільненим розумовим і фізичним розвитком, що страждає на часті блювання після прийому їжі. У сечі визначена фенілпіровиноградна кислота. Наслідком якого порушення є дана патологія?

A. *Обмін амінокислот

B. Фосфорно-кальцієвий обмін

C. Вуглеводний обмін

D. Ліпідний обмін

E. Водно-сольовий обмін

Патофізіологія клітини

  1. Під час вивчення студентами структури клітини, постало таке питання: "Що являють собою біологічні мембрани за своєю структурою"?

    1. *Бімолекулярний ліпідний шар з білковими компонентами

    2. Бімолекулярний ліпідний шар

    3. Мономолекулярний ліпідний шар

    4. Бімолекулярний білковий шар

    5. Бімолекулярний білковий шар з ліпідними компонентами

  2. Накопичення яких іонів у цитоплазмі м’язових клітин спричинює стійке перескорочення міофібрил ?

    1. *Кальцію.

    2. Натрію.

    3. Калію.

    4. Магнію.

    5. Водню.

  3. Що є безпосередньою причиною порушення видалення іонів кальцію з цитоплазми під час ушкодження клітини ?

    1. *Дефіцит АТФ.

    2. Ацидоз.

    3. Підвищення осмотичного тиску в цитоплазмі.

    4. Збільшення проникності клітинних мембран.

    5. Денатурація білків

  4. При синдромі реперфузії відбувається активація універсального пошкоджувального механізму мембранних структур, який має назву:

    1. *Пероксидне окиснення ліпідів

    2. beta -окиснення ліпідів

    3. Окиснення цитохромів

    4. Мікросомальне окиснення

    5. Цикл Кнопа-Лінена

  5. У сорокалітньої жінки з гіпертонічною хворобою при обстеженні лікар-кардіолог виявив на ЕКГ ознаки гіпертрофії лівого шлуночка. Загальний стан хворої задовільний, скарги відсутні. Ключовим механізмом, що забезпечує компенсацію серцевої недостатності в даному випадку є активація синтезу білків

    1. *Мітохондрій

    2. Саркоплазматичного ретикулуму

    3. Цитоплазми

    4. Цитолемальної мембрани

    5. Лізосом

  6. Хворий помер від інфаркту міокарда. Проведене патогістологічне дослідження міокарду виявило значні контрактурні зміни в кардіоміоцитах. Це зумовлено нагромадженням в кардіоміоцитах іонів

    1. *Кальцію

    2. Водню

    3. Натрію

    4. Магнію

    5. Хлору

  7. Експериментальній тварині ввели блокатор цитохромоксидази, що призвело до миттєвої загибелі. Яка з перелічених речовин може мати таку дію?

    1. *Ціанід калію

    2. Нітрит калію

    3. Сульфат калію

    4. Фосфат калію

    5. Оксалат калію

  8. При синдромі реперфузії активуються процеси вільнорадикального окиснення, що призводить до пошкодження клітинних мембран та порушення специфічних функцій клітин. Ці зміни пов’язані з надмірним накопиченням в цитоплазмі іонів:

    1. *Кальцію

    2. Магнію

    3. Хлору

    4. Натрію

    5. Калію

  9. Тварина отруїлася невідомою речовиною, що призвело до її миттєвої загибелі за рахунок окиснення цитохромів. Яка це речовина ?

    1. *Ціанід калію

    2. Хлорид калію

    3. Сульфат калію

    4. Оротат калію

    5. Перманганат калію

  10. При дослідженні сироватки крові хворого виявлене підвищення рівня аланінамінотрансферази (АЛТ) і аспартатамінотрансферази (АСТ). Які зміни в організмі на клітинному рівні можуть призвести до подібної ситуації?

    1. *Руйнування клітин

    2. Порушення функції енергозабезпечення клітин

    3. Порушення ферментних систем клітин

    4. Ушкодження генетичного апарату клітин

    5. Порушення міжклітинних контактів

  11. Після введення оубаїну (речовини, яка блокує К+, Na+ - залежну АТФ-азу) у експериментальної тварини на ЕКГ зареєстровано зміни, що свідчать про ушкодження кардіоміоцитів. Які молекулярні механізми мали вирішальне значення в даному випадку?

    1. *Електролітно-осмотичні

    2. Ліпідні

    3. Ацидотичні

    4. Кальцієві

    5. Протеїнові

  12. Причиною порушення фагоцитозу можуть бути якісні зміни фагоцитів. Зміни яких субклітинних структур найбільш характерні для синдрому “лінивих лейкоцитів”?

    1. *Мікрофіламентів

    2. Рецепторів до хемотаксинів та опсонінів

    3. Специфічного мембранного глікопротеїну (GP 110)

    4. Мікротрубочок

    5. Бактерицидних систем

  13. Встановлено, що причиною виникнення пневмонії у дитини є вірусна інфекція. Який механізм ушкодження клітин при цьому буде ведучим?

    1. *Нуклеїновий

    2. Протеїновий

    3. Електролітно—осмотичний

    4. Ліпідний

    5. Кальцієвий

  14. У фазі, що передує діастолічному розслабленню міокарду, різко знижується концентрація іонів кальцію у саркоплазмі і в період діастоли його практично немає у вільному стані. Які з нижче перерахованих структур беруть участь у акумуляції кальцію?

    1. *Мітохондрії

    2. Лізосоми

    3. Рибосоми

    4. Т-система

    5. Ядерця

  15. При інфаркті міокарда в плазмі крові хворих різко зростає активність аспартатамінотрансферази. Причиною цього явища є :

    1. *Пошкодження мембран кардіоміоцитів і збільшення виходу ферментів у кров.

    2. Стимуляція активності фермента гормонами

    3. Активація процесу синтезу амінокислот

    4. Сповільнення розпаду амінокислот

16. У хворого з стенокардією прийом нітрогліцерину призвів до відновлення кровопостачання міокарда и зняв больові відчуття в ділянці серця. Який зперерахованих внутріклітинних механізмів забезпечить відновлення енергопостачання клітини при пошкодженні?

A. *Посилення ресинтезу АТФ

B. Послаблення ресинтезу АТФ

C. Збільшення проникності мембран

D. Збільшення транспорту кисню всередині клітини

E. Посилення утворення РНК

17. При старінні організму в усіх видах зв’язано-сполучної тканини зменшується вміст води та співвідношення основна речовина/ волокно. За рахунок зростання вмісту якої речовини спостерігаються ці зміни ?

A. * Колаген

B. Кератин

C. Хондроїтін - сульфат

D. Еластин

E. Актин

18. Повторне переливання резус-додатньої крові резус-від’ємному пацієнту викликало гемоліз донорских еритроцитів в кровоносному руслі реципієнта. Що є ймовірною причиною гемолізу в даному випадку?

A. * Реакція антиген-антитіло

B. Механічні порушення

C. Осмотичні впливи

D. Фагоцитарна реакція

E. Матаболичні порушення мембран

19. Встановлено, що токсична дія ціанідів виявляється у гальмуванні клітинного дихання. Який органоїд клітини є чутливим до цих отрут?

A. *Мітохондрії

B. Рибосоми

C. Клітинний центр

D. Лізосоми

E. Комплекс Гольджі

20. Виділяють декілька груп молекулярних механізмів, які мають важливе значення в патогенезі ушкодження клітин, що сприяє розвитку патології. Які процеси забезпечують протеїнові механізми ушкодження:

A. *Пригнічення активності ферментів

B. Перекисне окислення ліпідів

C. Активація фосфоліпаз

D. Осмотичне розтягнення мембран

E. Ацидоз

21. У працівників хімічного підприємства часто спостерігаються гепатити і гепатози, де провідним механізмом ушкодження клітин печінки є активація перекисного окислення ліпідів. Вживання якого вітаміну можна рекомендувати з метою захисту?

A. * Вітаміну Е

B. Вітаміну С

C. Вітаміну Д

D. Вітаміну К

E. Вітаміну В1

22. Жінка 52 р., хвора на рак молочної залози, пройшла курс променевої терапії. Розмір пухлини зменшився. Який з наведених механізмів ушкодження клітини найбільш обумовлює ефективність променевої терапії?

A.*Утворення вільних радикалів

B. Гіпертермія

C. Лізис NK-клітинами

D. Тромбоз судин

E. Мутагенез

23. Експериментального кроля опромінили рентгенівськими променями. Який механізм викликав активацію перекисного окислення ліпідів у цьому випадку?

A. *Посилене утворення вільних радикалів

B. Дефіцит антиоксидантних ферментів

C. Гіповітаміноз Е

D. Порушення циклу Кребса

E. Дія детергентів

24. Виділяють декілька груп молекулярних механізмів, які мають важливе значення в патогенезі ушкодження клітин, що сприяє розвитку патології. Які процеси забезпечують протеїнові механізми ушкодження?

A. Осмотичне розтягнення мембран

B. *Пригнічення ферментів

C. Ацидоз

D. Перекисне окиснення ліпідів

E. Активація фосфоліпаз

25. У хворого на ішемічну хворобу серця відзначається гіпертрофія міокарда, тахікардія, зниження ХОК. Який з механізмів є провідним в ушкодженні кардіоміоцитів у даному випадку?

A. Збільшення числа  та -адренорецепторів

B. Дегідратація кардіоміоцитів

C. *Пошкодження специфічних мембранних насосів

D. Втрата Mg2+ кардіоміоцитами

E. Втрата Ca2+ кардіоміоцитами

26. Після ремонту автомобіля в закритому приміщенні при працюючому двигуні у чоловіка з'явилися задишка, запаморочення, акроціаноз, частота дихання 24-26/хв. Газовий склад крові: pO2- 60 мм рт.ст., pCO2- 30 мм рт.ст.; у крові наявний карбоксигемоглобін. Про який вид гіпоксії можна думати?

A. Циркуляторна

B. Респіраторна

C. Гіпоксична

D. Тканинна

E. *Гемічна

Патологія периферичного кровообігу

1. Хворому І., 45 років, з|із| діагнозом “цироз печінки, асцит”|вироблено,справлено| видалено|видобування| 5 літрів рідини з черевної порожнини, що викликало|спричинило| розвиток непритомного стану, як прояву|вияв| ішемії головного мозку. Яке порушення кровообігу в черевній порожнині має місце в даному випадку?

A. *Артеріальна гіперемія

B. Ішемія

C. Венозна гіперемія

D. Тромбоз

E. Емболія

2. Внутрішньовенне введення| хлориду ртуті експериментальній тварині викликало| утворення| пристінкового тромбу. Який патогенетичний| чинник|фактор| є основним| в|в,біля| розвитку| цього| патологічного процесу|?

A. *Ушкодження судинної стінки

B. Активація системи| зсідання крові

C. Активація адгезії тромбоцитів

D. Уповільнення течії крові

E. Зменшення активності антикоагулянтів

3. В експерименті у кролика| видалено| верхній шийний| ганглій| симпатичного стовбура|. На боці видалення| спостерігається почервоніння| шкіри та місцеве підвищення температури|. Яка форма порушень| периферичного кровообігу розвинулась| у кролика?|в,біля|

A. *Нейропаралітична артеріальна гіперемія

B. Нейротонічна артеріальна гіперемія

C. Метаболічна артеріальна гіперемія

D. Венозна гіперемія

E. Стаз|

4. Чоловіка 30 років укусила бджола|. На місці укусу відмічається набряк, гіперемія, підвищення температури|. Назвіть головний патогенетичний| чинник|фактор| запального набряку|.

A. * Підвищення проникності мікросудин

B. Підвищення осмотичного| тиску| в|в,біля| вогнищі запалення|

C. Зниження онкотичного| тиску| крові

D. Підвищення кров’яного| тиску| в капілярах

E. Порушення лімфовідтоку

5. Жінка 42 років з невралгією трійчастого нерва скаржиться| на періодичне почервоніння правої половини| обличчя| та шиї, відчуття місцевого підвищення температури| та шкірної чутливості. Ці явища| можна| пояснити| розвитком | артеріальної гіперемії наступного виду:

A. *Нейротонічної

B. Нейропаралітичної

C. Метаболічної

D. Робочої

E. Реактивної

6. Після вимушеного| швидкого| підняття водолаза з глибини| на поверхню| у|в,біля| нього| з’явилися| ознаки| кесонної хвороби| – біль в|в,біля| суглобах|,свербіння шкіри, мерехтіння в очах, затьмарення| свідомості. Яким видом|вид| ємболії | зумовлені вищевказані явища?

A. * Газовою

B. Повітряною

C. Жировою

D. Тканинною

E. Тромбоемболією

7. При моделюванні запалення| на брижі жаби| під мікроскопом спостерігали розширення| артеріальних судин|, прискорення| кровотоку, осьовий| пул |тік крові. Який вид артеріальної гіперемії виникає| при цьому|?

A. * Метаболічна

B. Вакатна

C. Нейропаралітична

D. Реактивна

E. Робоча

8. У|в,біля| хворого з|із| переломом гомілковостопного суглоба після|потім| зняття гіпсової пов'язки спостерігається набряк стопи, ціаноз, місцеве зниження температури|обсяг|. Який вид порушення кровообігу спостерігається при цьому?

A. *Венозная гіперемія

B. Робоча гіперемія

C. Метаболічна артеріальна гіперемія

D. Реактивна гіперемія

E. Ішемія

9. Хворому із закритим| переломом плечової кістки накладена гіпсова пов’язка|. Наступного дня з’явилася| припухлість, синюшність і місцеве зниження температури| кисті травмованої руки. Про який| розлад| периферичного кровообігу свідчать ці ознаки|?

A. *Венозна гіперемія

B. Артеріальна гіперемія

C. Ішемія

D. Тромбоз

E. Емболія

10. Під час гри| у|в,біля| волейбол спортсмен після| стрибка| приземлився| на зовнішній край стопи. Виник гострий| біль в|в,біля| гомілковостопному суглобі. Потім з'явилася| припухлість, шкіра почервоніла, стала теплішою на дотик|. Який вид розладу| периферичного кровообігу має місце| в даному| випадку|?

A. *Артеріальна гіперемія

B. Ішемія

C. Стаз|

D. Венозна гіперемія

E. Тромбоз

11. У|в,біля| хворого з тромбофлебітом нижніх кінцівок раптово| після навантаження| виникла| задишка|, різкий біль в|в,біля| грудях|, ціаноз, розширення| шийних| вен. Яке порушення| кровообігу розвинулося| у|в,біля| хворого?

A. *Тромбоемболія легеневої артерії

B. Тромбоемболія вінцевих судин|

C. Тромбоемболія судин| головного мозку|

D. Тромбоемболія мезентеріальних судин|

E. Тромбоемболія ворітної вени|

12. О.Б. Фохт, вивчаючи механізм порушення гемодинаміки у собак з емболією малого кола кровообігу, встановив, що в умовах ваготомії перебіг емболії полегшується, тривалість життя тварини продовжується. Яка ланка патогенезу при цьому виключається?

A.* Рефлекторне гостре зниження артеріального тиску

B.Різке падіння венозного тиску

C.Підвищення тиску у легеневій артерії

D.Зменшення хвилинного об’єму серця

E.Рефлекторне звуження легеневих артеріол

13. При аварійному підйомі з глибини| у водолаза виникли| судоми| із втратою| свідомості. Який патогенетичний| механізм є головним| в|в,біля| розвитку| цих| порушень|?

A. *Газова емболія

B. Гіпоксія

C. Токсична дія кисню|

D. Токсична дія азоту

E. Гіперкапнія

14. У|в,біля| хворої з переломом стегнової кістки виникла| емболія малого кола|кілок| кровообігу. Який вид емболії у|в,біля| хворої?

A. *Жирова

B. Тромбоемболія

C. Тканинна

D. Газова

E. Повітряна

15. У|в,біля| пілота на висоті 14000 м виникла| аварійна розгерметизація кабіни. Який з видів емболії у нього| розвинувся|?

A. *Газова

B. Тканинна

C. Тромбоемболія

D. Повітряна

E. Жирова

16. У|в,біля| хворого з пораненням| китиці почав утворюватися| набряк. В|в,біля| якій стадії порушення| місцевого кровообігу це| відбувається?

A. *Венозної гіперемії

B. Стазу|

C. Артеріальної гіперемії

D. Спазму артеріол

E. Престазу

17. У|в,біля| хворого з облітеруючим ендартеріїтом проведена гангліонарна симпатектомія. Який вид артеріальної гіперемії виник| в результаті операції?

A. *Нейропаралітична

B. Нейротонічна

C. Метаболічна

D. Робоча

E. Реактивна

18. У пацієнта з|із| хронічною серцевою|сердечний| недостатністю виявлено збільшення в'язкості крові, при капіляроскопії виявлено пошкодження|ушкодження| стінок судин|посудина| мікроциркуляторного| русла. Яке з|із| перерахованих порушень мікроциркуляції можливе в даному випадку?

A. * Сладж-феномен

B. Тромбоз

C. Емболія

D. Артеріальна гіперемія

E. Венозна гіперемія

19. У пацієнта через добу після введення туберкуліну на внутрішній поверхні предпліччя утворилася зона гіперемії 15 мм в діаметрі, гаряча на дотикі, при пальпації болюча, що розцінюється як додатня проба Манту. Який можливий патогенетичний механізм обумовив порушення периферичного кровообігу у пацієнта?

A.*Метаболічний

B.Постішемічний

C.Токсичний

D.Нейротонічний

E.Нейропаралітичний

20. Відомо, що самим частим у розвитку артеріальної гіперемії є міопаралітичний механізм, проте для можливого його попередження лікарю потрібно вирішити, що є передумовою цього механізму?

А.*Перевага міогенного тонусу в дрібних артеріях та артеріолах.

В. Підвищена продукція NO в зоні гіперемії.

С. Прискорення артеріального кровотоку.

D. Підвищений тонус симпатичної нервової системи.

E. -

21. У хворого 50 років, водія автобуса, розвинувся синдром гострого легеневого серця внаслідок тромбоемболії судин малого кола кровообігу. Яка найбільш імовірна причина даної тромбоемболії?

A.* Тромбоз вен нижніх кінцівок

B. Атеросклероз аорти

C. Аневризма аорти

D. Пневмоторакс

E. Рак печінки

22.Внаслідок ураження электричним струмом у постраждалого в ураженій кінцівці утворився тромб. Назвіть первинний механізм тромбоутворення в даному випадку :

A.*Пошкодження судинної стінки

B. Зміна фізико- хімічних властивостей крові

C. Сповільнення кровотоку

D. Збільшення активності зсідаючої системи крові

E. Зниження активності протизсідаючої системи крові

23. У хворого у післяопераційному періоді розвинувся сладж-синдром. Найбільш імовірною причиною його розвитку в даному випадку були розлади:

A.* Інтравазальні

B. Трансмуральні

C. Екстравазальні

D. Дифузійні

E .Фільтраційні

24. У пацієнта діагностовано тромбоемболію вен нижньої кінцівки. Прицьому спостерігається венозна гіперемія. Яка з перерахованих нижче ознак не характерна для цього розладу кровообігу

A.*Місцеве підвищення температури

B.Місцеве зниження температури

C. Синюшність

D. Набряк

E .Сповільнення кровотоку

25. При моделюванні запалення на брижі жаби під мікроскопом постерігали розширення переповнених кров’ю вен і капілярів, уповільнення руху крові, крайове стояння лейкоцитів. Який вид гіперемії виник при цьому?

A.*Венозна

B. Фізіологічна

C. Артеріальна

D. Реактивна

E. Робоча

26. Студент на екзамені не зміг вірно відповісти на питання екзаменаційного білету, що супроводжувалося почервонінням шкіри обличчя, відчуттям жару і невпевненістю поведінки. Який вид артеріальної гіперемії розвинувся у даному випадку?

A. Постішемічна

B. Метаболічна

C. Патологічна

D. Нейропаралітична

E. *Нейротонічна

Запалення

1. Через декілька годин після опіку в ділянці гіперемії та набряку шкіри у хворого з’явилося вогнище некрозу. Який механізм лежить в основі посилення руйнівних явищ в осередку запалення ?

A. *Вторинна альтерація

B. Первинна альтерація

C. Еміграція лімфоцитів

D. Діапедез еритроцитів

E. Проліферація фібробластів

2. У синтезі та виділенні медіаторів запалення приймають участь ряд клітин крові та сполучної тканини. В яких із цих клітин синтезується інтерлейкін-1?

A. *Макрофаги

B. Тканинні базофіли

C. Лімфоцити

D. Еозинофільні гранулоцити

E. Тромбоцити

3. Відомо, що медіатори запалення мають клітинне або гуморальне походження. Який з медіаторів, наведенних нижче, гуморального походження?

A.* Комплемент

B. Гістамін

C. Серотонін

D. Інтерлейкін-2

E. Інтерлейкін-1

4. Тварині, сенсибілізованій туберкуліном, внутрішньочеревинно введено туберкулін. Через 24 години при лапаратомії виявлено венозну гіперемію та набряк очеревини. У мазках-відбитках з очеревини велика кількість лімфоцитів та моноцитів. Який патологічний процес у тварини ?

A. * Алергічне запалення

B. Серозне запалення

C. Гнійне запалення

D. Фібринозне запалення

E. Асептичне запалення

5. При мікроскопічному дослідженні пунктату з осередка запалення у хворого з абсцесом шкіри знайдено велику кількість різних клітин крові. Які з цих клітин першими надходять із судин в тканини при запаленні?

A. * Нейтрофіли

B. Моноцити

C. Базофіли

D. Еозинофіли

E. Лімфоцити

6. Під час огляду шкіри лікар помітив у хворого патологічний процес у вигляді круглих підвищень червонуватого кольору, оточених зоною гіперемії. Які медіатори запалення зумовили явища судинної гіперемії?

A. *Гістамін

B. Інтерлейкін 1

C. Фактор активації тромбоцитів

D. Тромбоксан

E. Лізосомальні ферменти

7. Чоловік 30 років скаржиться на задуху, важкість у правій половині грудної клітки, загальну слабкість. Температура тіла 38,9'С. Об’єктивно: права половина грудної клітки відстає від лівої. Плевральна пункція виявила ексудат. Що являється провідним чинником ексудації у хворого?

A. *Підвищення проникності стінки судин

B. Підвищення кров’яного тиску

C. Гіпопротеїнемія

D. Агрегація еритроцитів

E. Зменшення резорбції плевральної рідини

8. Біохімічні зміни у вогнищі запалення - гіперосмія, гіперонкія, ацидоз, набряк. Розвиток гіперосмії, в деякій мірі, пов'язаний із зростанням концентрації К+ в зоні запалення. Вкажіть причини, що спричинили гіперкаліємію при запаленні.

A.* Інтенсивна деструкція пошкоджених клітин

B. Збільшення проникності судинної стінки

C. Активація проліферативних процесів

D. Пригнічення глікогенолізу в зоні запалення

9. Яким клітинам належить провідна роль в очищенні вогнища запалення від продуктів альтерації ?

A.* Моноцитам-макрофагам

B. Нейтрофільним гранулоцитам

C. Тканинним базофілам

D. Лімфоцитам

E. Фібробластам

10. При подагрі у хворих часто спостерігається збільшення та деформація суглобів внаслідок запалення. Який вид запалення лежить в основі цих змін?

A. * Проліферативне

B. Альтеративне

C. Ексудативне

D. Фібринозне

E. Змішане

11. Юнак 17 років захворів гостро, температура тіла підвищилася до 38,50С з’явився кашель, сльозотеча, виділення з носа. Яке запалення розвинулось у юнака ?

A. * Катаральне

B. Серозне

C. Фібринозне

D. Гнійне

E. Геморагічне

12. Який показник аналізу крові найбільш імовірно свідчить про наявність в організмі вогнища запалення ?

A. * Зростання кількості нейтрофільних лейкоцитів

B. Зростання кількості ретикулоцитів

C. Зростання кількості тромбоцитів

D. Зростання кількості базофільних лейкоцитів

E. Зниження кількості моноцитів

13. При мікроскопії мазка ексудату, отриманого від щура з асептичним перитонітом, з додаванням в ексудат пташиних еритроцитів, виявлені макрофаги, оточені чужорідними еритроцитами. Якій стадії фагоцитозу відповідає описана картина?

A. *Прилипання

B. Незавершеного фагоцитозу

C. Наближення

D. Занурення

E. Внутрішньоклітинного перетравлювання

14. При ревматизмі у хворих часто виявляється збільшення та деформація суглобів. Який вид запалення лежить в основі цих змін?

A.* Проліферативне

B. Ексудативне

C. Геморагічне

D. Альтеративне

E. Фібринозне

15. У хворого Д., 32 років, гнійна рана в нижній третині передпліччя. Хворому зроблено мазок із гнійного вмісту рани. Які клітини в основному виявлено у мазку при забарвленні за Романовським-Гімзою?

A. *Нейтрофіли

B. Еозинофіли

C. Лімфоцити

D. Еритроцити

E. Базофіли

16. До дерматолога звернулася пацієнтка зі скаргами на екзематозне ураження шкіри рук, що з’являється після контакту з миючим засобом “Лотос”. Використання гумових рукавичок запобігає цьому. Патологічна реакція шкіри зумовлена:

A. * Т-лімфоцитами

B. В-лімфоцитами

C. Моноцитами

D. Нейтрофілами

E. Базофілами

17. При моделюванні запалення нижньої кінцівки у тварини підвищилася температура тіла, збільшився вміст антитіл та лейкоцитів у крові. Які речовини зумовили розвиток цих загальних реакцій організму при запаленні?

A. * Інтерлейкіни

B. Глюкокортикоїди

C. Мінералокортикоїди

D. Лейкотрієни

E. Соматомедини

18. У хворого через добу після травми розпух колінний суглоб. При його пункції отримано 30 мл рідини рожевого кольору з питомою щільністю 1020. Загальний вміст білка в ній - 3%, альбумінів - 0,3%, глобулінів 2%, фібриногену - 0,7%, лейкоцитів - 1-3, еритроцитів - 15-20, місцями до 50 у полі зору. Якого характеру ексудат отриманий при пункції колінного суглоба у пацієнта?

A.* Геморагічний

B. Серозний

C. Гнійний

D. Гнильний

E. Фібринозний

19. Клітинний склад ексудату у вогнищі гострого запалення залежить переважно від:

A.* Природи флогогенного чинника

B. Ступеня проникності капілярів

C. Концентрації гістаміну в крові

D. Рівня гарячки

E. Порушень мікроциркуляції

20. Внаслідок необережності лаборантові у порожнину рота на язик потрапила соляна кислота. З якого процесу почнеться глосит у хворого?

A. * Альтерації

B. Порушення обміну речовин

C. Ексудації

D. Порушення мікроциркуляції

E. Артеріальної гіперемії

21. У хворого після травми виникло запалення. Це супроводжувалось підвищенням проникності судинної стінки для білка з розвитком набряку. Яка речовина викликала ці зміни?

A. *Гістамін

B. Адреналін

C. Ангіотензин

D. Вазопресин

E. Ацетилхолін

22. У хворого, що переніс травму з подальшим наступним геморагічним бурситом лівого колінного суглоба при огляді через 3 місяці відмічається обмеження об'єму рухів у даному суглобі внаслідок утворення рубця. Який компонент запалення є основою розвитку цього стану?

A. *Проліферація

B. Первинна альтерація

C. Вторинна альтерація

D. Ексудація

E. Порушення мікроциркуляції

23. При дослідженні запалення піддослідній тварині ввели смертельну дозу правцевого токсину в порожнину абсцесу, індукованого скипідаром. Але піддослідна тварина не загинула. Вкажіть найбільш ймовірну причину такого результату досліду?

A. *Формування бар'єру навколо запалення

B. Активація синтезу антитіл при запаленні

C. Стимуляція лейкопоезу при запаленні

D. Посилення васкуляризації місця запалення

E. Активація дезінтоксикаційної функції фагоцитів

24. При моделюванні запалення на брижі жаби спостерігали крайове стояння лейкоцитів та їх еміграцію крізь судинну стінку. Який із перелічених факторів обумовлює цей процес?

A. * Вплив хемотаксичних речовин

B. Збільшення онкотичного тиску в осередку запалення

C. Зниження онкотичного тиску в судинах

D. Збільшення гідростатичного тиску в судинах

E. Зменшення гідростатичного тиску в судинах

25. При мікроскопії препарату брижі жаби виявлено, що в деяких капілярах відмічається маятникоподібний рух крові, формені елементи при цьому (зокрема лейкоцити) з циркулюючого пулу виходять у пристінковий, а деякі навіть випускають псевдоподії в стінку капіляра. Якій стадії судинної реакції при запаленні відповідає описане явище?

A. *Престазу

B. Стазу

C. Короткочасного спазму судин

D. Артеріальній гіперемії

E. Венозній гіперемії

26. Вкажіть послідовність еміграції різних видів лейкоцитів в зону гострого гнійного запалення :

A.* Нейтрофіли, моноцити, лімфоцити

B. Лімфоцити, нейтрофіли, моноцити

C. Моноцити, нейтрофіли, лімфоцити

D. Моноцити, лімфоцити, нейтрофіли

E. Нейтрофіли, лімфоцити, моноцити

27. Головними проліферуючими клітинами, які формують грануляційну тканину в зоні запалення є:

A.* Фібробласти і ендотеліоцити капілярів

B. Лімфоцити і фібробласти

C. Ендотеліоцити капілярів і лаброцити

D. Лейкоцити і ендотеліоцити капілярів

E. Гістіоцити і фібробласти

28. У хворого з плевритом у плевральній порожнині виявлена зловонна рідина, яка містить у собі біогенні аміни, гази. Який різновид запалення в даному випадку?

A.* Гнильне

B. Альтеративне

C. Катаральне

D. Гнійне

E. Фібринозне

29. Після відриву меніска у спортсмена розвинулося запалення колінного суглоба. Який із патогенетичних факторів є головною ланкою патогенезу запалення?

A. *Пошкодження

B. Артеріальна гіперемія

C. Біль

D. Набряк

E. Венозна гіперемія

30. Жінку вкусив собака в ділянці правого литкового м'яза. При огляді: в зоні укусу спостерігається набряклість, припухлість тканин, гіперемія шкірних покривів. Який з перерахованих механізмів бере участь в розвитку артеріальної гіперемії при запаленні?

А. *Викид гістаміну

B. Здавлення венул набряклою тканиною

C. Зниження еластичності судинної стінки

D. Набухання ендотелію

E. Підвищення в'язкості крові

31. Хворий пред'являє скарги на сухий кашель, болі у м'язах, ломоту в суглобах, порушення апетиту, головний біль. Температура тіла - 40˚С, в аналізі крові: лейкоцитоз, збільшення ШОЕ, позитивна реакція на С-реактивний білок. Установлено діагноз: грип. Який провідний механізм розвитку первинної альтерації при вірусних інфекціях?

А. *Порушення реалізації генетичної програми клітини

B. Пошкодження клітинних мембран

С. Порушення механізмів енергозабезпечення клітини

D. Пошкодження клітинних рецепторів

E. Руйнування лізосом клітини

32. Клітинний склад ексудату значною мірою залежить від етіологічного фактору запалення. Які лейкоцити першими потрапляють у вогнище запалення, викликаного стафілококами?

A.* Нейтрофільні гранулоцити

B. Моноцити

C. Мієлоцити

D. Еозинофільні гранулоцити

E. Базофіли

33. В експерименті одержано алергічне запалення шкіри у тварини. Спостерігається гіперемія, набряк та еміграція лейкоцитів в осередок запалення. З дією якого медіатору пов’язаний хемотаксис лейкоцитів?

A. *Компоненти комплементу С3а, С5а

B. Гістамін

C. Серотонін

D. Тромбоксан

E. Еластаза

34. У дитини 3-х років розвинувся алергічний дерматит з везикульозними висипаннями на шкірі. Які клітини переважатимуть в ексудаті?

А. * Еозинофіли

B. Лімфоцити

C. Моноцити

D. Базофіли

E. Нейтрофіли

35. При хірургічному розкритті післяін'єкційного абсцесу одержано 2 мл гнійного ексудату. При мікроскопії в ньому знайдені поліморфноядерні лейкоцити. Який з перерахованих чинників виконує провідну роль в еміграції лейкоцитів у вогнищі запалення?

А. *Позитивний хемотаксис

В. Зміна проникності судин

С. Підвищення колоїдно-осмотичного тиску в вогнищі запалення

D. Зміна заряду лейкоцитів

E. Амебоподібні рухи лейкоцитів

36. У хворого з хронічним запальним процесом шкіри і підшкірної клітковини виявлено утворення надлишкових грануляцій. Для корекції лікування призначено глюкокортикоїди, що попереджають розвиток надлишкових грануляцій, оскільки вони:

A. * Мають катаболічний ефект

B. Прискорюють процеси мітозу

C. Сповільнюють дозрівання клітин

D. Мають анаболічний ефект

E. Зупиняють процеси старіння клітин

37. У відпочиваючого в санаторії, в результаті сонячного опіку, на шкірі спини утворилися міхурі заповнені світлою рідиною, оточені зоною гіперемії, болючі. Який з перерахованих механізмів лежить в основі формування стадії ексудації у вогнищі запалення?

А. *Збільшення колоїдно-осмотичного тиску в тканині

B. Зменшення виведення рідини внаслідок утворення бар'єру

С. Зменшення рівня кейлонів у тканині

D. Збільшення кількості лізосомальних ферментів

E. Еміграція лейкоцитів з судин

38. З метою гальмування розвитку проліферативного запалення у хворого, йому місцево в ділянку післяопераційного рубця призначили глюкокортикоїдну мазь. Який механізм гальмування глюкокортикоїдами процесів проліферації?

А. *Гальмування проліферації фібробластів

B. Активація проліферації макрофагів

С. Стимуляція резорбції колагену еозинофілами

D. Збільшення синтезу колагенових волокон

E. Активація коллагеназ, руйнуючих колаген

39. Хворий зі спадковим синдромом Чедіака – Хігасі страждає від гнійничкових уражень шкіри, часто хворіє на гнійний отит та тонзиліт. Дефект якої системи лежить в основі цього захворювання?

A.* Фагоцитозу

B. Комплементу

C. Клітинного імунітету

D. Гуморального імунітету

E. Клітинного та гуморального імунітету

40. Під час запалення спостерігається вихід лейкоцитів із судин у тканину. Які речовини стимулюють хемотаксис лейкоцитів у осередок запалення?

A.* Компоненти комплементу С3а, С5а

B. Серотонін

C. Імуноглобуліни

D. Гістамін

E. Кініни

41. У хворого, який часто хворіє на гнійні захворювання, виявлений спадковий дефіцит мієлопероксидази. Дефект якої системи лежить в основі цього захворювання?

A. *Фагоцитозу

B. Комплементу

C. Клітинного імунітету

D. Гуморального імунітету

E. Клітинного та гуморального імунітету

42. У експерименті на брижі жаби добре видно стадії судинної реакції при запаленні. Хто сформулював першу [судинну] теорію запалення?

A.* Конгейм

B. Вірхов

C. Шаде

D. Мечніков

E Менкін

43. Хворий з фурункулом скаржиться на сильний біль у руці, набряк та почервоніння шкіри, затруднення рухів, гарячку, сонливість. Який з цих симптомів відноситься до загальних проявів запалення?

A. *Гарячка

B. Біль

C. Набряк

D. Порушення функції

E. Почервоніння

44. У місці опіку з’явилася ділянка некрозу. Який фактор викликає первинну альтерацію?

A. *Флогогенний агент

B. Гіперосмія

C. Набряк

D. Медіатори запалення

E. Гіпоксія

45. У місці опіку з’явилося почервоніння. Які медіатори запалення викликають розширення артеріол?

A.* Гістамін, серотонін

B. Тромбоксани

C. Лізосомні ферменти

D. Інтерлейкіни

E. Фактори росту

46. У хворого з рваною раною кількість гною значно зменшилася, на дні рани з’явилися грануляції. Які біологічно активні речовини стимулюють проліферацію у вогнищі запалення?

A.*Фактори росту

B. α2-макроглобулін

C. Альбуміни

D. Гістамін

E. Простагландини

47. В ділянці панарицію відмічається гіперемія тканин, набряк, біль. Який механізм сприяє ексудації при запаленні?

A. *Підвищення гідростатичного тиску крові

B. Підвищення онкотичного тиску крові

C. Підвищення температури тіла

D. Зменшення проникності судинної стінки

E. Проліферація фібробластів

48. При запаленні ока у|в,біля| хворого відмічається накопичення| мутної рідини з високим| вмістом білка на дні передньої камери|, яке отримало| назву| - гіпопіон. Який процес| лежить| в основі вище вказаних змін?

A. *Порушення мікроциркуляції

B. Первинна альтерація

C. Вторинна альтерація

D. Проліферація

E. Репарація

Неспецифічна реактивність. Алергія

1. При дослідженні стану імунної системи хворого із хронічним грибковим ураженням шкіри виявлено порушення клітинного імунітету. Зменшення яких показників крові найбільш характерне при цьому ?

A. * Т-лімфоцитів

B. Імуноглобулінів G

C. Імуноглобулінів E

D. В-лімфоцитів

E. Плазмоцитів

2. У хворого П., після травми виникла необхідність введення протиправцевої сироватки, проте проба на чутливість до сироватки виявилася позитивною. Як провести специфічну гіпосенсибілізацію хворого? Введенням :

A. *Малих доз специфічного алергену

B. Фізіологічних доз глюкокортикоїдів

C. Розрішуючої дози специфічного алергену

D. Лікувальних доз антигістамінних препаратів

Е. Введення адреналіну

3. У новонароджених щуренят в експерименті була видалена загрудинна залоза. При цьому розвинулася хвороба, яка характеризується різким зниженням у крові лімфоцитів, розвитком інфекцій, спленомегалією, зупинкою росту і летальним наслідком. Яке порушення функції імунної системи при цьому спостерігається?

A. *Недостатність системи Т - лімфоцитів

B. Недостатність системи В - лімфоцитів

C. Гіперфункція системи Т - лімфоцитів

D. Гіперфункція системи В - лімфоцитів

E. Комбінований дефект Т - і В - лімфоцитів

4. У хворого через 9 діб після введення лікувальної сироватки з'явилася кропив'янка, свербіж та набряк шкіри і слизових оболонок, припухлість лімфатичних вузлів. Яка патологія має місце у хворого?

A.* Сироваткова хвороба

B. Феномен Шварцмана

C. Набряк Квінке

D. Феномен Овері

E. Поліноз

5. У хворого діагностована бронхіальна астма алергічної природи. Як провести специфічну гіпосенсибілізуючу терапію ?

A.* Малими дозами специфічного антигену

B. Антигістамінними препаратами

C. Малими дозами адреналіну

D. Глюкокортикоїдами

E. Мінералокортикоїдами

6. Чоловік 30 років травмував праве яєчко. Через 6 тижнів після завершення запалення з'явилася біль у лівому яєчку. Об'єктивно: шкіра в ділянці лівого яєчка гіперемована, набрякла, яєчко болюче на дотик, щільне. Який процес лежить в основі цього пошкодження ?

A. *Аутоалергія

B. Параалергія

C. Гетероалергія

D. Феномен Артюса

E. Феномен Овері

7. Хлопчик 1,5 років постійно хворіє на піодермію та тричі хворів на пневмонію. У крові пангіпогаммаглобулінемія. Який тип імунодефіциту виник у дитини?

A. * Гіпогаммаглобулінемія Брутона

B. Швейцарський тип

C. Гіпоплазія тимусу

D. Синдром Віскотта – Олдріча

E. Синдром Луї-Барр

8. Масугі викликав розвиток гломерулонефриту у щурів таким чином: гомогенат нирки щура вводив кролю. Через декілька тижнів сироватку сенсибілізованого кроля вводив щурам. Який тип алергічної реакції за Джеллом та Кумбсом лежить в основі розвитку гломерулонефриту у щурів?

A. * Цитотоксичний

B. Анафілактичний

C. Імунокомплексний

D. Гіперчутливість сповільненого типу

E. Стимулюючий

9. Заздалегідь сенсибілізованій морській свинці внутрішньовенно ввели 10 мл кінської сироватки. Через 30' відмічено: наїжування шерсті, тварина чихає, кашляє, чухає мордочку, дихання затруднене, судорожне, мимовільна дефекація і сечопуск. Яка стадія патогенезу анафілактичної реакції спостерігається у тварини?

A. *Патофізіологічна

B. Сенсибілізації

C. Патохімічна

D. Імунологічна

Е. Клінічних проявів

10. Жінка 27 років звернулася зі скаргами на свербіж та печію в очах, сльозотечу, чихання, виділення з носа. Симптоми з'явилися після поїздки влітку за місто. Діагностовано поліноз. Якого типу алергічна реакція розвинулася при цьому?

A.* Анафілактична

B. Цитотоксична

C. Реакція утворення імунних комплексів

D. Сповільненої гіпер чутливості

E. Стимулююча

11. Хворому на туберкульоз зроблено аналіз крові. Які зміни лейкоцитів найбільш характерні саме для туберкульозної реакції?

A. *Лімфоцитоз

B. Еозинофілія

C. Нейтрофільоз

D. Ядерний зсув лейкоцитарної формули вправо

E. Ядерний зсув лейкоцитарної формули вліво

12. У студентки, 20 років, третю весну підряд з початком цвітіння тополі починається свербіж і гіперемія очей, носа, ринорея, кашель, дрібне уртикарне висипання на відкритих ділянках тіла. При обстеженні виявлено різке збільшення рівня IgЕ. Алергологом призначена специфічна гіпосенсибілізуюча терапія. Вкажіть, яким чином проводять дане лікування?

A. *Багаторазовим введенням малих доз алергену

B. Одноразовим введенням великої дози алергену

C. Введенням антигістамінних препаратів

D. Введенням гормонів наднирників

E. Введенням хлористого кальцію

13. У ВІЛ-інфікованого хворого спостерігається пригнічення активності імунної системи. Ураження яких клітин найбільш ймовірно обумовлює стан імунодефіциту в цього хворого?

A. * Т-хелперів

B. Т-супресорів

C. Макрофагів

D. В-лімфоцитів

E. Т-кілерів

14. Яка ознака найбільш вірогідно характеризує наявність у хворого швейцарського типу імунодефіциту?

A.* Дефіцит Т- і В- лімфоцитів

B. Аутосомно-рецесивне успадкування

C. Лімфопенія

D. Зменшення плазматичних клітин

E. Гіпоплазія загрудинної залози

15. Жінку 44 років вжалила оса внаслідок чого розвинувся шок. В анамнезі вже була важка алергічна реакція на укус оси. Об’єктивно: пульс 179 уд /хв, слабкий, АТ-80/40 мм рт. ст., ЧД-26 за хв. Яка головна ланка патогенезу анафілактичного шоку?

A. *Зниження периферичного опору судин

B. Тахікардія

C. Біль

D. Зменшення ударного об'єму серця

E. Зменшення об’єму циркулюючої крові

16. У дитини двох років встановлено діагноз гіпоплазії тимуса. Який показник стану імунної системи є найбільш характерним для цього імунодефіциту?

A. * Зниження кількості Т-лімфоцитів

B. Зниження кількості В-лімфоцитів

C. Дефіцит Т і В-лімфоцитів

D. Відсутність плазматичних клітин

E. Зниження імуноглобулінів М

17. У хворого діагностовано тиреотоксикоз. В крові знайдено антитиреоїдні антитіла. Який тип алергічної реакції за Кумбсом і Джеллом спостерігається при розвитку цього захворювання?

A. * Стимулюючий

B. Анафілактичний

C. Цитотоксичний

D. Імунокомплексний

E. Гіперчутливість сповільненого типу

18. У хворого спостерігається синдром Ді Джорджі, в основі якого лежить гіпоплазія загрудинної залози До якої форми імунної патології відноситься це захворювання?

A.* Спадкового дефіциту Т-лімфоцитів

B. Спадкового дефіциту В- лімфоцитів

C. Набутого дефіциту В- лімфоцитів

D. Набутого дефіциту Т-лімфоцитів

E. Імунодепресії в системі Т- лімфоцитів

19. Для попередження відторгнення трансплантату після пересадки органів обов'язковим є проведення курсу гормонотерапії з метою імуносупресії. Які гормони застосовують при цьому?

A. *Глюкокортикоїди

B. Мінералокортикоїди

C. Статеві гормони

D. Катехоламіни

E. Щитовидної залози

20. Який із перерахованих патологічних процесів відноситься до реакцій негайного типу?

A.* Поліноз

B. Аутоалергія

C. Контактна алергія

D. Реакція відторгнення трансплантату

E. Бактеріальна алергія

21. Синдромом набутого імунодефіциту вважається пошкодження ВІЛ

A. *Т-хелперів

B. Т-супресорів

C. В-лімфоцитів

D. Макрофагів

E. Нейронів головного мозку

22. Хворому 44 років з гострою пневмонією призначили пеніцилін внутрішньом’язово. Після проведення ін’єкції стан хворого різко погіршився: з’явилася задишка, хворий покрився холодним потом. Пульс 140 за хв., слабкого наповнення. АТ 90/40 мм.рт.ст. Яке ускладнення найбільш ймовірно виникло у хворого?

A. *Анафілактичний шок

B. Тромбоемболія легеневої артерії

C. Кардіогенний шок

D. Інфекційно-токсичний шок

E. Тромбоз

23. До лікаря звернулася жінка зі скаргами на те, що у весняний період в неї з'являється хриплість голосу, почервоніння повік зі сльозотечею. Який тип алергічної реакції за Джеллом і Кумбсом розвивається у пацієнтки?

A. * Анафілактичний

B. Уповільненої гіперчутливості

C. Цитотоксичний

D. Імунокомплексний

E. Стимулюючий

24. У хворого з протеїнурією, гематурією, набряками і артеріальною гіпертензією встановлено діагноз “гострий гломерулонефрит ”. Назвіть алергічні реакції за Кумбсом і Джеллом, які у 80% випадків є причиною гломерулонефриту?

A.* Імунокомплексні

B. Анафілактичні

C. Цитотоксичні

D. Гіперчутливості сповільненого типу

E. Стимулюючого типу

25. Після введення великої дози антитіл у базальну мембрану клубочків нирок в піддослідної тварини розвинувся гострий гломерулонефрит. Який вид алергічної реакції за класифікацією Кумбса і Джелла лежить в основі цієї патології?

A. *Цитотоксичний

B. Анафілактичний

C. Імунокомплексний

D. Гіперчутливість сповільненого типу

E. Стимулюючий

26. У новонародженої дитини з судорожним синдромом і дефектом міжшлуночкової перегородки серця при рентгенологічному дослідженні грудної клітки виявлена гіпоплазія тимусу. Який імунодефіцит можна припустити в даного хворого?

A. *Синдром Ді Джорджі

B. Хвороба Брутона

C. Синдром Віскотта-Олдріча

D. Синдром Гуда

E. Атаксію-телеангіоектазію Луї-Барр

27. До лікувального закладу звернувся хворий зі скаргами на появу свербіння в носі, постійного чхання, задухи, кон’юнктивіту, які з’являються щовесни у період появи тополиного пуху. Про який тип алергічної реакції за Джеллом і Кумбсом йде мова?

A. *Анафілактичний (атопічний)

B. Цитотоксичний

C. Імунокомплексний

D. Клітинно-опосередкований

E. Стимулюючий

28. У хлопчика 5 років, як і у його дідуся, часто виникають пневмонії та гнійні ураження шкіри. При імунологічному обстеженні виявлено відсутність В-лімфоцитів. Яке порушення імунної системи має місце в даному випадку?

A. *Гіпогаммаглобулінемія Брутона

B. Гіпоплазія тимусу

C. Комбінований імунодефіцит

D. Синдром Шерешевського-Тернера

E. Синдром Дауна

29. Після введення місцевого анестетика у пацієнта розвинувся анафілактичний шок. Який механізм порушення кровообігу є провідним при цьому?

A. *Зменшення тонусу судин

B. Гіперволемія

C. Біль

D. Активація симпато-адреналової системи

E. Зниження скоротливої функції серця

30. Після вживання меду у підлітка з'явилася кропив'янка, що супроводжується лейкоцитозом. Який вид лейкоцитозу виник у даному випадку?

A. *Еозинофільний

B. Лімфоцитоз

C. Моноцитоз

D. Базофільний

E. Нейтрофільний

31. Чоловікові 27 років була проведена туберкулінова проба Манту. Через 24 години на місці ін'єкції відмічається інфільтрат 40х35 мм, шкіра над яким гіперемійована. Яка із груп біологічно активних речовин лежить в основі розвитку алергічного запалення у пацієнта?

A.* Лімфокіни

B. Кініни

C. Простагландини

D. Лейкотрієни

E. Біогенні аміни

32. Чоловік 25 років скаржиться на часто виникаючі запальні захворювання різної локалізації. Встановлено, що він - ін'єкційний наркоман. Проба на ВІЛ-інфекцію виявилася позитивною. Який з перерахованих типів клітин імунної системи найбільш суттєво вражається ВІЛ?

A.*Хелпери

B. Кілери

C. Нейтрофільні гранулоцити

D. Плазматичні клітини

Е. Еозинофіли

33. Після 4-го підшкірного введення кінської сироватки у кролика на стегні розвинулося гостре запалення за типом феномена Артюса. До якого виду зміненої реактивності відноситься така реакція?

A. * Гіперергія

B. Позитивна гіпоергія

C. Негативна гіпоергія

D. Дизергія

E. Анергія

34. Хворому М. з метою введення лікувальної дози протиправцевої сироватки було зроблено пробу на чутливість, яка виявилася позитивною. Як найбільш правильно провести специфічну гіпосенсибілізацію хворому? Введенням:

A. *Малих доз протиправцевої сироватки

B. Глюкокортикоїдів

C. Розрішуючої дози протиправцевої сироватки

D. Антигістамінних препаратів

E. Імунодепресантів

35. У мишей з відсутнім волосяним покривом (тобто nude - голі) не було клітинних реакцій уповільненого типу. Для цієї патології найбільш вірогідним є:

A.*Відсутність загрудинної залози

B. Відсутність гамаглобулінів у крові

C. Порушення гемопоезу

D. Дефект фагоцитозу

E. Дефіцит компонентів системи комплементу

36. Жінку 32 років вжалила оса. На шкірі лівої щоки (в місці укусу) - набряк та гіперемія. Який механізм набряку є первинним в даному випадку?

A. *Підвищення проникності капілярів

B. Підвищення гідростатичного тиску крові у капілярах

C. Зниження онкотичного тиску крові

D. Підвищення онкотичного тиску тканинної рідини

E. Утруднення лімфовідтоку

37. Морській свинці ввели з метою сенсибілізації 0,1мл кінської сироватки. Якими зовнішніми ознаками проявляється стан сенсибілізації?

A. *Немає зовнішніх проявів

B. Висипи на шкірі

C. Припухлість суглобів

D. Підвищення температури тіла

E. Біль

38. У медсестри маніпуляційного кабінету із стажем роботи 20 років розвинувся контактний дерматит верхніх кінцівок. До якого типу імунних порушень відноситься дане захворювання?

A. *Алергічна реакція сповільненого типу

B. Первинний імунодефіцит

C. Алергічна реакція негайного типу

D. В-клітинний імунодефіцит

E. Т-клітинний імунодефіцит

39. Через декілька хвилин після проведення лікарем-стоматологом місцевої анестезії зуба новокаїном у пацієнта раптово з'явилася слабкість, свербіж шкіри. Об'єктивно відмічається гіперемія шкірних покривів, тахікардія, зниження артеріального тиску до 70/40 мм рт.ст.. До якого типу алергічних реакцій відноситься описана патологія?

A. *Анафілактичних

B. Цитотоксичних

C. Стимулюючих

D. Клітинно-опосередкованих

E. Імунокомплексних

40. Після тижневого застосування нового косметичного засобу в жінки розвинулося запалення повік з гіперемією, інфільтрацією і болючістю. Алергічна реакція якого типу має місце у пацієнтки?

A. * ІV

B. I

C. II

D. III

E. V

41. Хворий Д., 15 років поступив в алергологічне відділення з діагнозом бронхіальна астма. Надмірне утворення яких антитіл викликає розвиток основних клінічних симптомів?

A. *IgE

B. IGA

C. Ig G

D. IGM

E. IGD

42. Хворому Н., 18 років був виставлений діагноз цукровий діабет I типу, інсулінозалежний. Який тип алергічних реакцій зумовлює пошкодження бета-клітин?

A. *Цитотоксичний

B. Анафілактичний

C. Імунокомплексний

D. Гіперчутливості сповільненого типу

E. Стимулюючий

43. Дівчинка 10 років під час гри порізала ногу уламком скла і була направлена в поліклініку для введення протиправцевої сироватки. З метою попередження розвитку анафілактичного шоку лікувальну сироватку вводили по Безредку. Який механізм лежить в основі даного способу гіпосенсибілізації організму?

A. *Зв'язування фіксованих на тканинних базофілах IgE

B. Блокування синтезу клітинних медіаторів

C. Стимуляція імунологічної толерантності до антигена

D. Стимуляція синтезу антиген-специфічних IgG2

E. Зв'язування рецепторів до IgE на тканинних базофілах

44. У хворого Б., аналіз крові виявив ознаки ВІЛ- інфекції. Ушкодження яких імунокомпетентних клітин характерно для СНІДу?

A. *Т-хелперів

B. Т-кілерів

C. В-лімфоцитів

D. Макрофагів

E. Нейтрофілів

45. Пацієнт попередив хірурга-стоматолога, що застосування знеболюючих препаратів може викликати алергічний шок. Збільшення кількості в крові якого біогенного аміну може бути причиною такого стану?

A.*Гістаміну

B. ГАМК

C. Серотоніну

D. Дофаміну

E. Триптофану

46. На 8 день після введення протиправцевої сироватки з приводу інфікованої рани стопи в пацієнта підвищилася температура до 38˚С, з'явилися болі в суглобах, висипка, свербіж. З боку крові спостерігається лейкопенія і тромбоцитопенія. Який тип алергічної реакції розвинувся у хворого?

A. *Імунокомплексна

B. Анафілактична

C. Цитотоксична

D. Гіперчутливість сповільненого типу

E. Стимулююча

47. В основі розвитку імунних і алергічних реакцій організмом використовуються однакові механізми відповіді імунної системи на антиген. Визначте основну відмінність алергічних реакцій від імунних?

A. *Развиток пошкодження тканин

B. Кількість потрапляючого антигена

C. Особливість будови антигенів

D. Шляхи попадання антигенів в організм

E. Спадкова схильність

48. У хворого при обстеженні виявили тромбоцитопенію, що виникла внаслідок підвищеного руйнування тромбоцитів. Який механізм розвитку цієї тромбоцитопенії?

A. *Імунний

B. Мієлотоксичний

C. Дефіцитний

D. Дисрегуляційний

E. Підвищеного споживання

49. У 20-річної дівчини, яка страждає на поліпоз кишечника, в анамнезі часті грибкові та вірусні захворювання. Недостатність якого компоненту імунної системи є найбільш вірогідною в даному випадку?

A. *Т-лімфоцитів

B. В-лімфоцитів

C. Натуральних кілерів

D. Комплементу

E. Фагоцитів

50. У 12-річного хлопця часто виникають вірусні та бактеріальні інфекції, екзематозні ураження шкіри. При обстеженні виявлено зменшення Т-лімфоцитів та IgМ при нормальному вмісті IgA і IgG. Який вид патології імунної системи спостерігається у дитини?

A. *Комбінований імунодефіцит

B. Гіпоплазія тимуса

C. Гіпогаммаглобулінемія Брутона

D. Синдром Шерешевського-Тернера

E. Спадковий дефіцит системи комплементу

51. Хворий звернувся до лікаря зі скаргами на те, що кожної весни, в період цвітіння рослин у нього відмічаються головний біль, нежить, слабкість, підвищення температури. Який тип алергічної реакції за Джеллом і Кумбсом в даному випадку?

A. *Анафілактичний

B. Цитотоксичний

C. Стимулюючий

D. Імунокомплексний

E. Клітинно-опосередкований

52. У дівчинки 6 років набряк повік, губ, шиї, слизової оболонки язика виникл після того, як вона з'їла апельсин. Раніше на апельсини виникали висипання на шкірі, свербіж. Який патогенетичний механізм являється ведучим у розвитку набряку в дівчинки?

A.* Підвищення проникності капілярів

B. Порушення лімфовідтоку

C. Підвищення онкотичного тиску тканинної рідини

D. Зниження онкотичного тиску крові

E. Підвищення гідростатичного тиску крові у капілярах

53. Хвора 27 років закапала в очі краплі, до складу яких входив пеніцилін. Через декілька хвилин з’явилися зуд та печія тіла, набряк губ, повік, кашель зі свистом, став падати АТ. Які імуноглобуліни приймають участь в розвитку даної алергічної реакції?

A. *IgE і IgG.

B. IgM і IgG.

C. IgA і IgM

D. IgM i IgD

E. IgG i IgD

54. Через 5-8 днів після застосування значних кількостей лікувальної сироватки у хворого виявилися шкірні висипання, зуд, припухлість, болі в суглобах, підвищилася температура тіла, в сечі з”явився білок. Був поставлений діагноз сироваткова хвороба Що є головною ланкою в патогенезі цього синдрому?

A. * Накопичення в крові циркулюючих імунних комплексів

B. Дегрануляція тканинних базофілів

C. Активація Т-кілерів

D. Активація макрофагів

E. Цитоліз формених елементів крові

55. У хворого А. в серпні після роботи на дачі развинувся стан, що характеризувався лікарем, як стан підвищеної і якісно зміненої реакції на надходження в організм сполук антигенної або гаптенової природи. Який з перерахованих станів найбільш підходить під описану лікарем характеристику?

A. *Алергія

B. Анафілаксія

C. Параалергія

D. Тахіфілаксія

E. Імунологічна толерантність

56. У пацієнта, який хворіє на бронхіальну астму, що має інфекційно - алергічну природу, лаборант виявив порушення в лейкоцитарній формулі. Кількість яких форм лейкоцитів при цьому змінилася?

A. *Еозинофілів

B. Нейтрофілів

C. Лімфоцитів

D. Моноцитів

E. Базофілів

57. У хворої, яка страждає на поліноз, після поїздки за місто з'явилися набряки губ і повік, сльозотеча, виділення з носа, печія в очах. Який основний механізм розвитку цього набряку?

A. * Підвищення проникності капілярів

B. Підвищення онкотичного тиску крові

C. Підвищення онкотичного тиску тканин

D. Підвищення гідростатичного тиску в капілярах

E. Порушення лімфовідтоку

58. Хлопчик 10 років скаржиться на свербіж, пекучість шкіри, набряк слизової порожнини носа та верхніх дихальних шляхів. Приїхав з дачі 30 хв. тому, де їв мед. Куди віднести дану реакцію?

A. *Алергічна реакція негайного типу

B. Алергічна реакція сповільненого типу.

C. Первинний імунодефіцит.

D. Загальна анафілаксія.

E. Імунодефіцит швейцарського типу.

59. Батьки 5 річного хлопчика на прийомі у лікаря відмічають неодноразові простудні захворювання, які закінчуються розвитком пневмонії, є гнійні висипання на шкірі. При лабораторному обстеженні виявлено: відсутність всіх типів імуноглобулінів та відсутність В-лімфоцитів в пунктаті лімфовузлів. Яке порушення імунної системи має місце?

A. *Гіпогамаглобулінемія Брутона

B. Імунодефіцит швейцарського типу

C. Гіпопластична анемія

D. Агранулоцитоз

E. Синдром Луі Барр

60. Дитині після аналізу імунограмми поставили діагноз первинний імунодефіцит гуморальної ланки імунітету. Яка з причин може привести до розвитку первинного імунодефіциту в організмі дитини?

А. *Спадкові порушення в імунній системі

B. Порушення в процесі ембріонального розвитку

С. Порушення обміну речовин в організмі матері

D. Порушення реактивності і резистентності організму

E. Токсичне пошкодження В-лімфоцитов

61. З метою пластики післяопікових рубців, хворому провели пересадку клаптя здорової шкіри від донора, що привело до формування клітинної імунної відповіді на антиген. Що утворюється в організмі при формуванні клітинної імунної відповіді?

А. *Сенсибілізовані лімфоцити

B. Специфічні імуноглобуліни

С. Активовані макрофаги

D. Клітини імунологічної пам'яті

E. Медіатори (гістамін, серотонін)

62. У хворого виявлена аутоімунна гемолітична анемія яка розвивається по цитотоксичному типу. Які речовини є антигенами при алергічних реакціях II типу?

А. *Модифіковані рецептори клітинних мембран

B. Антибіотики

С. Гормони

D. Сироваткові білки

E. Модулятори запалення

63. Для моделювання анафілактичного шоку у морської свинки провели пасивну сенсибілізацію. Що слід ввести з метою пасивної сенсибілізації?

A.*Специфічні імуноглобуліни.

B. Кінську сироватку

C. Сенсибілізовані Т- лімфоцити

D. Тканинні базофіли

E. В-лімфоцити

64. Через деякий час після пересадки нирки у хворого відбулося відторгнення трансплантату. Які сенсибілізовані імунні ефектори спричинили таке явище?

A. *Т- лімфоцити

B. В-лімфоцити

C. ІgG

D. Іg А

E. Іg Е

65. Під час проведення хірургічних маніпуляцій було використано новокаїн з метою знеболення. Через 10хв. у хворого з’явилися блідість шкірних покривів, задишка, гіпотензія. Алергічну реакцію якого типу можна запідозрити?

A. * Анафілактичного

B. Цитотоксичного

C. Імунокомплексного

D. Стимулюючого

E. Клітинно-опосередкованого

66. Внутрішні бар’єри регулюють надходження з крові в органи і тканини потрібних речовин і перешкоджають проникненню токсичних речовин. Назвіть речовини, які можуть підвищити проникність цих бар’єрів.

A. *Кініни

B. Катехоламіни

C. Солі кальцію

D. Флавоноїди

E. Снотворні засоби

67. У дівчинки 5 років спостерігається важкий перебіг бактеріальних інфекцій. При дослідженні сироватки крові – значне зменшення концентрації IgG (5 мкмоль/л), IgA (0,5 мкмоль/л), IgM (0,02 мкмоль/л) та відсутність В-лімфоцитів. Який розвинувся імунодефіцит?

A.* Хвороба Брутона

B. Швейцарський тип агамаглобулінемії

С. Дисімуноглобулінемія

D. Синдром Ді Джорджі

E. Синдром Луї-Барр

68. У дитини 8 років спостерігаються неврологічні порушення (атаксія мозочкового типу) та патологічне розширення кровоносних судин кон’юктиви та шкіри (телеангіоектазії). Діагностовано первинний імунодефіцит – синдром Луї-Барр. Відсутність яких імуноглобулінів буде спостерігатись в крові дитини?

A. *Ig A

B. Ig E

C. Ig D

D. Ig M

E. Ig G

69. У хворого виявлено злоякісне новоутворення. Зниження активності яких клітин імунної системи можна припустити у цьому випадку?

A. *Т-лімфоцитів

B. В-лімфоцитів

C. Плазмобластів

D. Плазмоцитів

E. Стовбурних клітин кісткового мозку

70. При обстеженні хворих пародонтитом відмічається залежність ступеня пошкодження тканин пародонта від кількості лізоциму в слині та ясневій рідині. Показник якого захисту організму при цьому досліджується?

A.* Неспецифічна резистентність

B. Гуморальний імунітет

C. Клітинний імунітет

D. Аутореактивність

E. Толерантність

71. Хворий звернувся до лікаря з приводу частих і тривалих інфекцій, в основному викликаних гнійпродукуючою флорою. В результаті обстеження виявлений синдром Чедіака-Хігаші. Яка функція лейкоцитів порушена при цьому захворюванні?

А. *Хемотаксис

B. Фагоцитоз

C. Піноцитоз

D. Адгезія

E. Активація

72. Після проведення проби на чутливість до антибіотика пеніциліну у хворого розвинувся анафілактичний шок, що супроводжується зниженням АТ, бронхоспазмом, набряком Квінке, кропив'янкою. Причиною цього стану є:

А. * Розширення капілярних судин венозного відділу кров'яного русла

B. Активація симпато-адреналової системи

C. Зниження скоротливої функції серця

D. Порушення функції нирок

E. Біль

73. У хворого з грибковим ураженням шкіри виявлено порушення клітинного імунітету. Зниження яких показників найбільш характерні при цьому?

A. * Т-лімфоцитів

B. Імуноглобулінів G

C. Імуноглобулінів E

D. В-лімфоцитів

E. Плазмоцитів

74. Після пересадки нирки хворому проведено курс гормонотерапії. Які гормони були застосовані?

A. *Глюкокортикоїди

B. Мінералокортикоїди

C. Катехоламіни

D. Статеві

E. Тиреоїдні

75. У резус- від’ємної вагітної жінки можливий імунний конфлікт між матір’ю та плодом. Наявність яких з перерахованих імуноглобулінів може привести до цього явища?

A.* Ig G

B. Ig М

C. Ig А

D. Ig Е

E. Ig Д

76. У хворої при обстеженні діагностовано аутоімунний тіреотоксикоз. Для цього захворювання характерний такий тип алергічної реакції за Кумбсом і Джеллом:

A. *Стимулюючий

B. Цитотоксичний

C. Імунокомплексний

D. Анафілактичний

E. Клітинно-опосередкований

77. Чоловік 25 р. був ужалений бджолою. При наданні першої медичної допомоги йому було введено адреналін. Для зупинки розвитку якого з наведених нижче імунологічних механізмів була проведена маніпуляція?

A.*Генералізованої анафілаксії

B. Вивільнення інтерлейкінів з макрофагів

C. Зв’язування з антирецепторними антитілами

D. Місцевого утворення імунних комплексів

E. Активації системи комплементу

78. В патогенезі якого з перелічених процесів не бере участь комплемент?

A. *Контактний дерматит

B. Феномен Артюса

C. -

D. Гемолітична трансфузійна реакція

E. Сироваткова хвороба

79. Аутоімунний інсуліт, що є причиною розвитку цукрового діабету 1 типу, розвивається за механізмом аутоімунних реакцій 4-го типу, за Кумбсом і Джеллом. Що спричинює пошкодження В-клітин острівців підшлункової залози?

A.* Т-лімфоцити

B. IgE

C. IgA

D. В-лімфоцити

E. Імунні комплекси

80. При базедовій хворобі тироцити продукують надмірну кількість гормонів Т3 і Т4. Що спричинює дисфункцію тироцитів при цьому аутоімунному захворюванні?

A. *IgG

B. IgA

C. IgE

D. Імунні комплекси

Е. Т-лімфоцити

81. У дитини вживання в їжу червоної риби завжди супроводжується відчуттям дискомфорту, болями в животі, іноді проносом. Як називається локальна реакція анафілактичного типу у травному тракті?

A. *Алергічний гастроентерит

B. Кропив'янка

C. Алергічний нежить

D. Поліноз

E. Анафілактичний шок

82. При контакті з сухим кормом для рибок, дитина миттєво починає задихатися. Вкажіть клінічний прояв алергічної реакції анафілактичного типу в нижніх дихальних шляхах:

A.* Анафілактична бронхіальна астма

B. Кропив'янка

C. Анафілактичний шок

D. Алергічний нежить

E. Поліноз

83. Пацієнт С. хворіє на поліноз. Що є головною ланкою патогенезу цього алергічного захворювання?

A. *Алерген– індукована дегрануляція тканинних базофілів

B. Преципітація імунних комплексів у місці їх формування

C. Циркуляція імунних комплексів

D. Алерген – індукована активація фагоцитів

E. Втрата толерантності до антигену

84. Влітку, під час цвітіння амброзії, деякі люди страждають від місцевих анафілактичних реакцій на цю рослину. Як називаються клінічні прояви шкірних анафілактичних реакцій?

A. *Кропив'янка

B. Алергічний нежить

C. Поліноз

D. Анафілактична бронхіальна астма

E. Анафілактичний шок

85. У дитини алергія на волоський горіх. Вживання в їжу будь-якого продукту, що містить горіхи, призводить до появи у дитини висипу, що свербить. З якими антитілами пов'язаний розвиток алергічних реакцій анафілактичного типу?

A.* IgE

B. IgM

C. IgG

D. IgA

E. IgD

86. У місцях розвитку анафілактичних реакцій виражені гіперемія, набряк, біль. Який медіатор анафілаксії визначає розвиток вищезгаданих розладів?

A.* Гістамін

B. Гепарин

C. Фактори хемотаксису

D. Фактор активації тромбоцитів

E. Білки комплементу

87. Відразу після введення препарату у пацієнта з'явилися задуха, відчуття страху, зниження артеріального тиску. Імунні реакції якого типу, за Кумбсом і Джеллом, лежать в основі цього стану у хворого?

A.* Анафілактичні

B. Гуморальні цитотоксичні

C. Імунокомплексні

D. Клітинні цитотоксичні

E. Стимулюючі

88. Боксер М., 30 років, звернувся до лікаря зі скаргами на головні болі, зниження пам’яті, сповільнення рухів, підвищення тонусу м’язів, тремтіння пальців рук, нижньої щелепи. Встановлено діагноз: “Паркінсонічний синдром”. За яким механізмом виникло ушкодження головного мозку у хворого?

A. * Демаскування аутоаантигенів головного мозку внаслідок ушкодження ГЕБ

B. Порушення бар’єрної функції плазматичних мембран нейроцитів

C. Посилення протеолізу нервової тканини

D. Внутрішньоклітинний ацидоз

E. Токсичний вплив катехоламінів

89. На повторне введення протиправцевої сироватки у людини виникла важка алергічна реакція.Чим зумовлена ця реакція?

A. * Підвищеною чутливістю до сироватки

B. Підвищеною чутливістю до токсину правця

C. Підвищеною чутливістю до збудника правця

D. Формуванням стійкості до токсинів

Е. Антибіотикочутливістю

90. Дитина народилася з вовчою пащею. При обстеженні виявлено вади аорти, в крові – зменшення Т-лімфоцитів. Який імунодефіцитний синдром у новонародженого?

A. * Ді Джорджі

B. Віскотта – Олдріча

C. Чедіака – Хігасі

D. Луї – Барр

E. Швейцарський тип

Патологія тканинного росту.

Пухлини

1. Епідеміологічне дослідження поширення пухлин виявило високу кореляцію розвитку пухлин легень із табакокурінням. З дією якого хімічного канцерогену найбільш ймовірне виникнення даного виду патології?

A. *3,4-бензпірену

B. Ортоаміноазотолуолу

C. Афлатоксину

D. Метилхолантрену

E. Діетилнітрозаміну

2. Жінці виставлено діагноз ерозія шийки матки, яка є передпухлинною патологією. Який захисний механізм може попередити розвиток пухлини ?

A. * Збільшення природних кілерів (NK-клітин)

B. Високодозова імунологічна толерантність

C. Збільшення активності лізосомальних ферментів

D. Спрощення антигенного складу тканин

E. Низькодозова імунологічна толерантність

3. Чоловік 63 років, страждає раком стравоходу, метастази в лімфатичні вузли середостіння, ракова кахексія. Яка патогенетична стадія пухлинного процесу у чоловіка?

A. *Прогресії

B. Промоції

C. Трансформації

D. Ініціації

E. Катаплазії

4. Жінка 67 років, страждає раком шлунку з метастазами в печінку. Яка властивість пухлинних клітин зумовлює їх здатність до метастазування ?

A. *Інфільтративний ріст

B. Біохімічний атипізм

C. Швидкий ріст

D. Автономність

E. Антигенна анаплазія

5. Чоловік 58 років, страждає раком сечового міхура. У процесі трудової діяльності мав контакт з канцерогенними речовинами. Дія якого з нижче перелічених канцерогенів найбільш вірогідна в даному випадку?

A.* В - нафтиламін

B. 20 - метилхолантрен

C. Бензпірен

D. Диметиламіноазобензол

E. Ортоаміноазотолуол

6. Проводиться медичний огляд робітників цеху по виробництву анілінових барвників. Наявність пухлини якої локалізації може бути оцінене як професійне захворювання, внаслідок контакту з бета-нафтиламіном?

A.* Сечового міхура

B. Стравоходу

C. Печінки

D. Нирок

E. Легень

7. У шлунку людини із неканцерогенних речовин в присутності соляної кислоти можливе утворення сполук, здатних викликати пухлинний ріст. До якої групи канцерогенів відносять ці сполуки?

A. *Нітрозаміни

B. Поліциклічні ароматичні вуглеводні

C. Аміни

D. Аміноазосполуки

E. Афлатоксин

8. Для пухлинної тканини характерна біохімічна анаплазія. Особливе місце в біохімії пухлин займає обмін вуглеводів і вироблення енергії. Назвіть особливості вуглеводного обміну в пухлинній тканині.

A. *Негативний ефект Пастера

B. Позитивний ефект Пастера

C. Ослаблення гліколізу

D. Негативний ефект Кребтрі

E. Пригнічення гліколітичних ферментів

9. Встановлено, що при розвитку гепатоми в ній часто припиняється синтез жовчних кислот. Про який вид анаплазії це свідчить?

A. *Функціональну

B. Енергетичну

C. Морфологічну

D. Біохімічну

E. Фізико- хімічну

10. Піддослідним тваринам з їжею давали нітрит натрію. У 80% тварин розвинулася пухлина. До якої групи канцерогенів відноситься дана сполука?

A. *Нітрозаміни

B. Аміноазосполуки

C. Поліциклічні ароматичні вуглеводні

D. Прості хімічні речовини

E. Гормони

11. Хворий на рак печінки довгий час контактував з диметиламіноазобензолом (ДАБ ). Чим пояснюється виникнення пухлини саме в цьому органі?

A.* Органотропністю ДАБ

B. Токсичною дією ДАБ

C. Порушенням жовчовиділення

D. Порушення жовчоутворення

E. Місцевою канцерогенною дією ДАБ

12. Чоловік тривалий час працює в нафтопереробній промисловості. Який з наведених класів канцерогенів зустрічається в його оточенні?

A.* Поліциклічні ароматичні вуглеводні

B. Аміноазосполуки

C. Нітрозаміни

D. Канцерогени біологічного походження

E. Ароматичні аміни

13. У 1910 р. Раус в експерименті отримав саркому курей шляхом введення їм безклітинного фільтрату, одержаного з саркоми курки. Який метод експериментального моделювання використовував автор?

A. *Індукції

B. Експлантації

C. Ізотрансплантації

D. Гомотрансплантації

E. Гетеротрансплантації

14. У хворого на операції знайшли пухлину шлунка з проростанням в слизову, підслизову і серозну оболонки. Виявлено метастази в перигастральні лімфовузли, віддалені метастази відсутні. Встановлено 3 стадію (T3, N1, M0) розвитку пухлини. Який етап розвитку патогенезу пухлини має місце в даному випадку?

А. *Пухлинної прогресії

B. Промоції

C. Перетворення протоонкогена в онкоген

D. Утворення онкобілків

E. Трансформації

15. Згідно з даними літератури, пухлинні процеси бувають у тварин, людей, риб. Чому пухлинний процес вважають загальнобіологічним явищем?

A. *Тому що пухлинний процес знаходять у різних біологічних різновидів, перерахованих в умові задачі

В. Тому що пухлини знаходять у комах

С. Тому що пухлини знаходять у риб

D. Тому що пухлини знаходять у ссавців

Е. Тому що пухлини знаходять у рептилій

16. Відомо, що в процесі адаптації до змін навколишнього середовища в організмі відбувається інтенсифікація обміну речовин з гіпербіотичними змінами тканинного росту, до яких відносяться:

А. *Усі перелічені

B. Гіпертрофія

С. Гіперплазія

D. Регенерація

E. Пухлини

17. Відсутність у пухлинних клітин ліміту Хейфліка було відкрито при дослідженні поділу клітин в культурі тканини. Який експериментальний метод вивчення пухлин було застосовано?

A. *Експлантації

B. Трансплантації

C. Індукції радіацією

D. Індукції хімічними канцерогенами

E. Індукції вірусами

18. У хворого виявлена злоякісна пухлина ока – ретинобластома. Яке генетичне порушення стало причиною виникнення цієї пухлини?

A. *Втрата репресора клітинного поділу

B. Транслокація протоонкогенів

C. Ампліфікація протоонкогенів

D. Мутація в протоонкогенах

E. Вбудовування вірусної ДНК в геном клітини

19. Біохімічний аналіз пухлини, вилученої у хворого, виявив інтенсивний гліколіз навіть в умовах достатнього надходження кисню до клітини. Така особливість обміну речовин пухлинної клітини є проявом:

A.* Біохімічної анаплазії

B. Морфологічної анаплазії

C. Пухлинної прогресії

D. Фізико-хімічної анаплазії

E. Ліміту Хейфліка

20. У хворого із злоякісною пухлиною легень виявлені новоутворення іншої локалізації. Це є наслідком:

A. *Метастазування

B. Експансивного росту

C. Інфільтративного росту

D. Анаплазії

E. Метаплазії

21. В експериментальної тварини змоделювали рак підшлункової залози. Які молекулярні механізми призводять до пухлинної трансформації?

A. * Гальмування генів-репресорів

B. Гальмування аденілатциклази

C. Активація протоонкогенів

D. Гальмування протоонкогенів

E. Активація генів-репресорів

22. У первинній карциномі легень внаслідок депресії синтезу речовин знайдено синтез гонадотропіну, що в нормі не утворюється в даній тканині. З якою особливістю пухлинної тканини це зв'язано?

А. *Метаплазією

B. Фізико- хімічною анаплазією

С. Функціональною анаплазією

D. Клітинним атипізмом

E. Антигенними особливостями

23. У хворого К., 46 років, діагностована злоякісна пухлина. Назвіть послідовність етапів процесу метастазування?

A. *Інтравазація,дисемінація,екстравазація

B. Дисемінація,екстравазація,інтравазація

C. Дисемінація,інтравазація,екстравазація

D. Екстравазація,дисемінація,інтравазація

E. Інтравазація,поглинання,дисемінація

24. У жінки при профілактичному огляді знайшли пухлину молочної залози із збільшенням пахвових лімфовузлів. Результати біопсії підтвердили наявність злоякісної пухлини. Який основний механізм інфільтративного росту злоякісної пухлини?

А. *Порушення контактного гальмування

B. Здатність пухлинних клітин до амебоїдного руху

С. Збільшення адгезивності пухлинних клітин

D. Підвищення активності кейлонів у клітині

E. Підвищення ферментативної активності лізосом

25. При обстеженні хворого з виразками порожньої кишки виявлено злоякісну пухлину підшлункової залози. Пухлинні клітини хворого продукують надвисокі кількості гастрину. Як називається така властивість деяких пухлин?

A. *Метаплазія

B. Анаплазія

C. Наближення до ембріонального стану

D. Малігнізація

E. Трансформація

26. На шкіру тварини наносили мікродози метилхолантрену, а пізніше змащували ту саму ділянку шкіри кротоновою олією, що призводило до виникнення пухлин. Яка стадія канцерогенезу пов’язана з дією кротонової олії?

A. *Промоція

B. Прогресія

C. Трансформація

D. Транслокація

E. Ініціація

27. Після третього курсу хіміотерапії у хворого із злоякісною пухлиною легень відсутні будь-які позитивні зміни. З якою властивістю пухлин пов’язаний розвиток їх резистентності до хіміотерапії?

A.* Прогресія

B. Промоція

C. Автономний ріст

D. Інвазивний ріст

E. Експансивний ріст

28. У курців канцерогени тютюнового диму і смол спричинюють пухлини дихальних шляхів. Який механізм трансформуючої дії хімічних канцерогенів?

A. *Взаємодія з ДНК

B. Взаємодія з РНК

C. Взаємодія з онкобілками

D. Дія на мітохондрії

E. Дія на клітинну мембрану

29. У постраждалого після опромінення виник гострий лейкоз. Який механізм трансформації характерний для фізичних канцерогенів?

A.* Мутаційний

B. Епігеномний

C. Трансплантаційний

D. Індукційний

E. Продукційний

30. У хворого на хронічний мієлолейкоз на фоні різкого зменшення чутливості пухлини до хіміотерапії виник бластний криз, що було розцінено лікарем як прояв пухлинної прогресії. Що складає суть пухлинної прогресії?

A.* Малігнізація пухлини

B. Метастазування пухлини

С. Антигенне спрощення

D. Автономність росту

E. Безмежність росту

31. Встановлено, що при витраті однієї і тієї ж кількості глюкози пухлинна тканина отримує енергії в 20-25 разів менше, ніж здорова. Про яку зміну в метаболізмі глюкози в пухлині говорить дане явище?

A. *Посилення анаеробного гліколізу

B. Посилення окислювальних процесів

C. Нормальне співвідношення цих процесів

D. Посилення тканинного дихання

E. Ослаблення анаеробного дихання

Патологія обміну речовин

1. У вагітної жінки розвинувся токсикоз з важкою повторною блювотою протягом доби. До кінця доби почали виявлятися тетанічні судоми і зневоднення організму. Яке порушення КЛР спричинило описані зміни?

A. *Негазовий алкалоз

B. Газовий алкалоз

C. Газовий ацидоз

D. Негазовий метаболічний ацидоз

E. Негазовий ацидоз

2. Хворий цукровим діабетом поступив у клініку зі скаргами на погіршення стану: загальну слабкість, поліурію, спрагу, загальмованість і сонливість. Відмічається дихання Куссмауля, аритмія серця, запах ацетону в повітрі, що видихається. Яке порушення КЛР викликало описану симптоматику?

A. *Негазовий метаболічний ацидоз

B. Газовий ацидоз

C. Газовий алкалоз

D. Негазовий алкалоз

E. Негазовий ацидоз

3. При підйомі в гори у альпініста розвинулися ейфорія, головний біль, запаморочення, серцебиття, задишка, яка чергувалася з апноє. Яке порушення кислотно-лужної равноваги розвинулося у альпініста?

A. *Газовий алкалоз

В. Метаболічний алкалоз

C. Негазовий алкалоз

D. Газовий ацидоз

E. Негазовий ацидоз

4. У хлопчика М., 2-ох років була діагностована хвороба Гірке, що супроводжується надмірним відкладанням глікогену в печінці і нирках та гіпоглікемією. При біохімічному дослідженні крові буде виявлено:

A. * Дефіцит глюкозо-6-фосфатази

B. Зниження активності аміло-1,6-глюкозидази

C. Дефіцит фруктозо-дифосфатальдолази

D. Зниження активності фосфорилази

E. Дефіцит кетогексокінази

5. У хворого з цукровим діабетом виявляється високий рівень гіперглікемії, кетонурія, глюкозурія, гіперстенурія і поліурія. Яка форма порушення кислотно-лужної рівноваги має місце в даній ситуації?

A. *Метаболічний ацидоз

B. Газовий ацидоз

C. Метаболічний алкалоз

D. Газовий алкалоз

E. Видільний алкалоз

6. У хворого 40 років після перенесеної травми головного мозку стало спостерігатися підвищене виділення сечі (до 8 - 10 л/добу) і спрага. Аналіз сечі показав відсутність глюкозурії і низьку відносну щільність. Яка форма порушення водно-сольового обміну можлива в період описаного захворювання, що не компенсується?

A. *Гіперосмолярна гіпогідратація

B. Ізоосмолярна гіпогідратація

C. Гіпоосмолярна гіпогідратація

D. Ізоосмолярна гіпергідратація

E. Гіпоосмолярна гіпергідратація

7. Тварина знаходиться в стані повного голодування. Основний обмін підвищений. Дихальний коефіцієнт дорівнює 1, азотистий баланс негативний. У якому періоді голодування перебуває тварина?

A. *Збудження

B. Пригнічення

C. Байдужості

D. Паралічів

E. Термінальному

8.У дитини з розумовою відсталістю встановлено зелене забарвлення сечі після додавання 5% розчину FeCl3. Про порушення обміну якої амінокислоти свідчить позитивний результат цієї діагностичної проби ?

A. * Фенілаланіну

B. Аргініну

C. Глутаміну

D. Тирозину

E. Триптофану

9. Через кілька тижнів після народження у дитини почали відмічатися прояви ураження ЦНС, шкіра та волосся посвітліли. При додаванні до свіжої сечі 5% розчину трихлороцтового заліза сеча забарвлюється в оливково-зеленений колір. Який найбільш вірогідний діагноз?

A.*Фенілкетонурія

B. Алкаптонурія

C. Фруктозурія

D. Тирозиноз

E. Альбінізм

10. У породілі через 24 години після пологів діагностовано емболію легеневої артерії. Виникла задишка з частим, глибоким диханням. Артеріальний тиск – 90/40 мм рт.ст., частота серцевих скорочень – 99/хв. Газовий аналіз: рН = 7,48; РСО2 =32 мм рт.ст.; [HCO3] = 20 ммоль/л; [BE] = +1ммоль/л. Яке з перерахованих нижче порушень кислотно-лужної рівноваги розвинулося у хворої?

A. *Декомпенсований респіраторний алкалоз

B. Компенсований респіраторний алкалоз

C. Декомпенсований метаболічний ацидоз

D. Компенсований метаболічний ацидоз

E. Респіраторний алкалоз + метаболічний ацидоз

11. Хвора, 24 р., скаржиться на сухість в роті, зниження маси тіла, незважаючи на підвищений апетит, часте сечовиділення. Яке дослідження для встановлення діагнозу необхідно призначити в першу чергу?

A. *Визначення рівня цукру в добовій кількості сечі

B. Аналіз сечі по Зимницькому

C. Загальний аналіз сечі

D. Дослідження білкових фракцій сироватки крові

E. Коагулограму

12. Чоловік 60 років страждає атеросклерозом судин. Яка з приведених нижче сполук відіграє провідну роль в патогенезі даного захворювання?

A.* Ліпопротеїни низької щільності

B. Ліпопротеїни високої щільності

C. Хіломікрони

D. Тканинний фермент ліпопротеїнкіназа

E. Комплекс жирних кислот з альбумінами

13. У дитини з пілоростенозом, який супроводжується частою блювотою, з”явилися ознаки зневоднення організму. Яка форма порушення кислотно-лужної рівноваги може розвинутися в даному випадку?

A. *Негазовий алкалоз

B. Негазовий ацидоз

C. Газовий ацидоз

D. Газовий алкалоз

E. Метаболічний ацидоз

14. При голодуванні (у другому його періоді) у хворого може підвищуватися кількість ліпідів у крові і спостерігатися гіпопротеїнемія. Яка форма гіперліпідемії має місце в даному випадку?

A. *Транспортна

B. Ретенційна

C. Аліментарна

D. Харчова

E. Перерозподільна

15. У хворого визначається наявність гіперглікемії, поліурії, гіперстенурії та глюкозурії. Для якої нозологічної форми патології обміну характерне таке поєднання цих клінічних проявів?

A. *Цукровий діабет

B. Нирковий діабет

C. Нецукровий діабет

D. Глікогеноз

E. Диенцефальне ожиріння

16. У хворого 30 років, діагностовано ішемічну хворобу серця. Приступи стенокардії турбують вже 3 роки. Встановлено спадковий характер захворювання. Який вид гіперліпопротеїнемії найбільш імовірно буде виявлений у пацієнта?

A.* ІІ А (гіпербеталіпопротеїнемія)

B. І (гіперхіломікронемія)

C. ІІІ (дисбеталіпопротеїнемія)

D. IV (гіперпребеталіпопротеїнемія)

E. V (гіперпребеталіпопротеїнемія і гіперхіломікронемія)

17. У вагітної жінки розвинувся токсикоз з важким повторним блюванням протягом доби. До кінця доби почали виявлятися тетанічні судоми і зневоднення організму. Яке порушення КЛ рівноваги спричинило описані прояви?

A.* Метаболічний алкалоз

B. Газовий алкалоз

C. Газовий ацидоз

D. Метаболічний ацидоз

E. Ацидоз виділення

18. У дитини 1,5 років періодичні напади слабкості натще серця, неоноразові втрати свідомості, клонічні судоми. Дитина низького зросту з худими кінцівками і великим животом, різко збільшена печінка. Пункційна біопсія виявила значне накопичення глікогену в гепатоцитах та в лейкоцитах. Що лежить в основі захворювання?

A. *Дефект печінкової глюкозо- 6-фосфатази

B. Дефект мальтази

C. Дефект аміло-1,6-глюкозидази

D. Дефект фосфоліпази

E. Дефект гексокінази

19. У хворого з гіперфункцією щитовидної залози підвищена температура тіла. Яке порушення енергетичного обміну є головним у підвищенні температури при цьому?

A.* Роз’єднання окиснення та окисного фосфорування

B. Збільшення розпаду глікогену

C. Посилення ліполізу

D. Активація ферментів у циклі Кребса

E. Активація ферментів дихального ланцюга

20. На 10 добу лікувального голодування хворий відчув збудження, з'явилося глибоке, шумне дихання, артеріальний тиск знизився до 90/60 мм рт.ст., зменшилося виділення сечі, сеча з запахом ацетону. Чим зумовлені описані явища?

A. *Кетозом

B. Негазовим алкалозом

C. Гіперглікемією

D. Гіпоглікемією

E. Газовим ацидозом

21. У дитини 3 років, яка страждає на квашіоркор, спостерігається порушення ороговіння епідермісу та збільшення його злущення, має місце жирова інфільтрація печінки. Який тип голодування спостерігається при цьому ?

A. *Білкове

B. Вуглеводне

C. Мінеральне

D. Енергетичне

E. Жирове

22. До лікаря звернувся чоловік 65 років зі скаргами на гострий біль у великих пальцях ніг. Він любить м’ясо та часто вживає пиво. Виникла підозра на подагру. Для підтвердження діагнозу вміст якої із перелічених речовин необхідно визначити в крові?

A. *Сечової кислоти

B. Сечовини

C. Лактату

D. Білірубіну

E. Кетонових тіл

23. Алкаптонурія виникає внаслідок блокади метаболізму тирозину на етапі:

A. *Перетворення гомогентизинової кислоти на малеїтацетооцтову

B. Перетворення гідроксифенілпіровиноградної кислоти на гомогентизинову

C. Перетворення малеїтацетооцтової кислоти на фумарилацетооцтову

D. Перетворення тирозину в меланін

E. Йодування тирозину вільним йодом

24. В клініку доставлений чоловік 30 років, з профузним проносом протягом 12 годин. Блювоти не було. Які зміни водно-електролітного балансу та кислотно-лужної рівноваги спостерігаються у чоловіка?

A. *Негазовий ацидоз з дегідратацією

B. Газовий ацидоз з дегідратацією

C. Газовий алкалоз з дегідратацією

D. pH крові без змін

E. Негазовий алкалоз з дегідратацією

25. Чоловік 64 років скаржиться на задишку, часте серцебиття, швидку втомлюваність. Ввечері з'являються набряки на нижніх кінцівках. Що із нижче переліченого являється патогенетичним чинником цих набряків?

A.* Підвищення гідростатичного тиску крові в венозних капілярах

B. Зниження онкотичного тиску крові

C. Підвищення онкотичного тиску тканинної рідини

D. Підвищення проникності капілярів

E. Порушення лімфовідтоку

26. У хворого на цукровий діабет знижені процеси регенерації, довго не загоюються рани. З якими змінами в обміні речовин це пов’ язане ?

A. * Пригніченням протеосинтезу

B. Накопиченням кетонових тіл

C. Ацидозом

D. Зменшенням надходження глюкози в клітини

E. Порушенням жирового обміну

27. Відомо, що фенілкетонурія виникає внаслідок мутації гена, що відповідає за перетворення амінокислоти фенілаланіну і розпаду її до кінцевих продуктів обміну – СО2 і Н2О. Вкажіть, який шлях обміну фенілаланіну приведе до розвитку фенілкетонурії.

A. *Фенілаланін - фенілпіруват - кетокислоти

B. Фенілаланін - тирозин - тироксин

C. Фенілаланін - тирозин - меланін

D. Фенілаланін - тирозин - норадреналін

E. Фенілаланін - тирозин - алкаптон

28. У хворого з дихальною недостатністю рН крові 7,35. Визначення рСО2 показало наявність гіперкапнії. При дослідженні рН сечі відмічається підвищення її кислотності. Яка форма порушення кислотно-лужної рівноваги в даному випадку?

A. *Компенсований газовий ацидоз

B. Компенсований метаболічний ацидоз

C. Декомпенсованний метаболічний ацидоз

D. Компенсований газовий алкалоз

E. Декомпенсований газовий алкалоз

29. Жінка 37 років скаржиться на загальну слабкість, часті запаморочення, ускладнене ковтання їжі, бажання їсти крейду. Шкіра та видимі слизові оболонки бліді. У крові: Ер. 3,4●1012/л, Hb 70 г/л, КП 0,7, ретикулоцити – 0,1%, лейк. 4,7●1012/л, еоз. – 2%, п./яд -3%, сегм.-64%, лімф. – 26%, мон. – 5%. ШЗЕ – 15 мм/г. Сироваткове залізо – 7,3 мкмоль/л. Дефіцит якої речовини зумовив виникнення захворювання?

A. *Заліза

B. Білка

C. Вітаміну В6

D. Вітаміну В12

E. Фолієвої кислоти

30. У хворого А., 18 років після перенесеної краснухи виникла втрата маси тіла, постійне відчуття спраги, підвищення апетиту. Об'єктивно: добова кількість сечі 6 л, глюкоза крові 17,8 ммоль/л, в сечі виявлено глюкозу та ацетон. Яку патологію можна запідозрити у хворого?

A. *Інсулінозалежний цукровий діабет

B. Нецукровій діабет

C. Стероїдний діабет

D. Інсулінонезалежний цукровий діабет

E. Діабетична нефропатія

31. Хворий В. 67 років, страждає атеросклерозом судин серця і головного мозку. При обстеженні виявлена гіперліпідемія. Який клас ліпопротеїнів плазми крові має найбільше значення в патогенезі атеросклерозу?

A. *Ліпопротеїни низької щільності

B. Хіломікрони

C. Альфа-ліпопротеїни

D. Ліпопротеїни високої щільності

E. Ліпопротеїни дуже низької щільності

32. Увiддiлення реанiмацiї доставлено непритомного пацiєнта з запахом ацетону з рота. Методи експрес-аналiзу виявили в кровi 17,3 ммоль/л глюкози. Пiдвищення вмiсту яких речовин призвело до втрати свiдомостi?

A. *Кетоновi тiла

B. Молочна кислота

C. Глюкоза

D. Жирнi кислоти

E. Сечовина

33. У хворого з цукровим діабетом виникла спрага, дисфагія та порушення психічної діяльності. Який тип розладів водно-електролітного балансу характеризує поява вказаних ознак?

A. *Дегідратація гіперосмотична

B. Дегідратація гіпоосмотична

C. Дегідратація ізоосмотична

D. Гіпергідратація гіпоосмотична

E. Гіпергідратація ізоосмотична

34. При обстеженні хворого визначається наявність гіперглікемії, кетонурії, поліурії, гіперстенурії і глюкозурії. Яка форма порушення кислотно-лужної рівноваги має місце в даній ситуації?

A.*Метаболічний ацидоз

B. Газовий ацидоз

C. Газовий алкалоз

D. Метаболічний алкалоз

E. Негазовий алкалоз

35. У відділення невідкладної допомоги з ливарного цеху доставлений хворий 23 років з масою тіла 60 кг Дослідження водного обміну показали: загальна вода 33 л (55% маси тіла), позаклітинний сектор 17,2 л (28,6%), внутрішньосудинна рідина 2,4 л (4%), внутрішньоклітинний сектор 15,8 л (26,3%). Р осм. - 340 мосм/л, Na+ - 160 ммоль/л. Діурез 0,4 л/доб.. Визначте вид дисгідрії.

A.*Гіперосмолярна гіпогідратація

B. Гіперосмолярна гіпергідратація

C. Ізоосмолярная гіпергідратація

D. Гіпоосмолярная гіпергідратація

E. Дисгідрії немає

36. У мишей із спадковим ожирінням встановлено гіперглікемію та зниження кількості інсулінових рецепторів у ліпоцитах. Який патогенетичний механізм є первинним в посиленні ліпогенезу у цих тварин ?

A. *Гіперінсулінемія

B. Гіпоінсулінізм

C. Гіпертрофія ліпоцитів

D. Зниження толерантності до глюкози

E. Підвищення депонування жирів

37. У людини, яка тривалий час голодувала, розвинулися набряки. Який основний механізм виникнення цих набряків?

A. *Зменшення онкотичного тиску крові

B. Збільшення онкотичного тиску тканин

C. Збільшення гідростатичного тиску венозної крові

D. Зменшення гідростатичного тиску тканин

E. Зменшення об’єму циркулюючої крові

38. Хворий з цукровим діабетом вчасно не отримав ін’єкцію інсуліну, що призвело до розвитку гіперглікемічної коми (вміст глюкози в крові – 50 ммоль/л). Який механізм є головним в розвитку цієї патології?

A. *Гіперосмія

B. Гіпокаліємія

C. Гіпоксія

D. Гіпонатріємія

E. Ацидоз

39. Хворий Л., 48 років, з цукровим діабетом поступив в лікарню у важкому прекоматозному стані. При дослідженні КЛР виявлено метаболічний ацидоз. Який первинний механізм зумовив виявлені зміни КЛР?

A. *Утворення недоокислених продуктів

B. Порушення використання О2 в клітинах

C. Порушення буферних систем крові

D. Виведення лужних компонентів з сечею

E. Зниження виведення СО2

40.У хворого на цукровий діабет внаслідок накопичення оксимасляної та ацетооцтової кислот має місце порушення кислотно-лужного стану, яке називається:

A. *Метаболічний ацидоз

B. Метаболічний алкалоз

C. Газовий ацидоз

D. Газовий алкалоз

E. Негазовий алкалоз

41. Собака знаходилася в термостаті при t 40˚С. У неї спостерігалося значне збільшення частоти дихання. Який вид порушення водно-електролітного обміну виник при цьому?

A. *Дегідратація гіперосмолярна

B. Дегідратація гіпоосмолярна

C. Дегідратація ізоосмолярна

D. Позитивний водний баланс

Е. Негативний водний баланс

42. У щура після 48 годин голодування з водою основний обмін зменшився на 20%, дихальний коефіцієнт 0,7, наявна ліпемія, негативний азотистий баланс. Для якого періоду повного голодування типові дані зрушення?

A. *Максимального пристосування

B. Неекономної витрати енергії

C. Термінального

D. Паралічів і парезів

E. Збудження

43. У щура після 72 годин голодування з водою маса тіла та основний обмін знизилися на 40%, дихальний коефіцієнт 0,8, має місце кетонемія, ацидоз, набряки. Для якого періоду повного голодування характерні дані зрушення?

A. *Термінального

B. Неекономного використання енергії

C. Максимального пристосування

Д. Збудження

Е. Байдужості

44. На 10-ту годину голодування з водою у щура визначається підвищення основного обміну, дихальний коефіцієнт складає 1,0, у крові збільшений вміст глюкокортикоїдів, катехоламінів. У сечі підвищений вміст азоту. Для якого періоду повного голодування типові дані зрушення?

A. *Неекономної трати енергії

B. Максимального пристосування

C. Термінального

Д. Тривоги

Е.Релаксації

45. У експериментальних щурів, що тривалий час отримували лише вуглеводну їжу, спостерігається накопичення води в тканинах. Який патогенетичний механізм є головним у розвитку набряку в даному випадку?

A. *Гіпоонкотичний

B. Мембраногенний

C. Дисрегуляційний

D. Лімфогенний

E. Гіперосмолярний

46. У підлітка 12 років, який протягом 3 місяців схуднув на 7 кг, вміст глюкози в крові становить 20 ммоль/л, несподівано розвинулася кома. Який вид цукрового діабету найбільш ймовірний в даному випадку?

A. *Інсулінозалежний (I тип)

B. Інсулінонезалежний (II тип)

C. Гіпофізарний

D. Стероїдний

E. Гіпертиреоїдний

47. Людина може протриматися без їжі протягом 40-60 діб. Яка з перерахованих субстанцій може бути перетворена на глюкозу для забезпечення енергетичних потреб головного мозку в цей термін голодування ?

A. * Амінокислоти

B. Ацетоацетат

C. Ацетон

D. Жирні кислоти

E. Бета-гідроксібутират

48. У хворого з патологією серцево-судинної системи розвинулися набряки на нижніх кінцівках. Який механізм розвитку серцевого набряку?

A. *Підвищення гідростатичного тиску у венозному кінці капіляру

B. Підвищення онкотичного тиску

C. Підвищення гідростатичного тиску у артеріальному кінці капіляру

D. Зниження осмотичного тиску

E. Порушення лімфовідтоку

49. В ув’язненого, який оголосив голодування, розвинулися набряки. Який механізм їх виникнення?

A. *Зменшення онкотичного тиску крові

B. Збільшення гідростатичного тиску венозної крові

C. Зменшення гідростатичного тиску тканин

D. Зменшення ОЦК

E. Збільшення онкотичного тиску тканин

50. Пацієнта на дачі ужалила бджола. При огляді: в ділянці укууа бджоли на лівій руці місцеве підвищення температури, гіперемія, набряк, великий червоний міхур. Який з механізмів є ведучим у розвитку набряку?

А. *Підвищення проникності судин

B. Зниження кровонаповнення судин

C. Пошкодження судин при укусі

D. Зниження онкотичного тиску в тканині

E. Зниження осмотичного тиску в тканині

51. У хворого з діагнозом `Цукровий діабет` при подальшому обстеженні встановлено порушення тесту толерантності до глюкози. Про що свідчить дане порушення?

А. *Обмеження надходження глюкози в інсулінзалежні клітини

B. Про посилення секреції глюкагону альфа-клітинами

C. Про гальмування секреції інсуліну бета-клітинами

D. Про збільшення інсулінорезистентності тканин

E. Про посилення секреції контрінсулярних гормонів

52. Голодування приводить до виникнення набряків. Яка причина цього явища?

A. *Зниження онкотичного тиску плазми

B. Гіповітаміноз

C. Порушення функції нирок

D. Серцева недостатність

E. Гіповолемія

53. При обстеженні у хворого виявлено глюкозурію при нормальному вмісті глюкози в крові. Яка можлива причина цього явища?

A. *Генетична недостатність транспорту глюкози в проксимальних канальцях

B. Гіперпродукція глюкагону

C. Гіпертиреоз

D. Гіперпродукція глюкокортикоїдів

E. Гіпотиреоз

54. При клінічному обстеженні у хворого виявлено гіперглікемію і глюкозурію. Яка ймовірна причина цього явища?

A. *Інсулінова недостатність

B. Порушення білкового обміну

C. Порушення ліпідного обміну

D. Посилений глюконеогенез

E. Посилений гліколіз

55. У жінки з ендокринною патологією виявлено підвищення основного обміну на 15%. Збільшення продукції якого гормону може викликати такий стан?

A. * Трийодтироніну

B. Глюкагону

C. Тиреокальцитоніну

D. Соматостатину

E. Альдостерону

56. Повне голодування проходить в своєму розвитку три періоди. Для третього (термінального) періоду голодування є характерним:

A. *Посилення розпаду білків життєвоважливих органів

B. Активація ліполізу в жировій тканині

C. Посилення катаболізму білків у м’язах та глюконеогенезу в печінці

D. Підвищення утворення кетонових тіл в печінці

E. Розвиток негазового ацидозу

57. У хворого, який страждає на діарею, в артеріальній крові рН становить 7,25, рСО2 – 34 мм рт.ст. Що може бути первинним в ускладненні його стану?

A. *Метаболічний ацидоз

B. Метаболічний алкалоз

C. Дихальний ацидоз

D. Дихальний алкалоз

E. -

58. Дівчина 9 років госпіталізована у відділення з діагнозом цукровий діабет І типу. При лабораторному обстеженні виявлено високий рівень кетонових тіл. Який основний механізм розвитку захворювання?

A. * Інсулінова недостатність

B. Інсулінова надмірність

C. Надмірність глюкагону

D. Надмірність соматостатину

E. Порушення комплексування інсуліну з рецепторами

59. Пацієнт 55 років звернувся до лікаря зі скаргами на часті судоми. Встановлено, що тривалий час він працює у гарячому цеху в умовах високих температур та зниженої вологості. Порушення якого виду обміну привело до цього стану?

A. *Водно - сольового

B. Вітамінного

C. Ліпідного

D. Білкового

E. Вуглеводного

60. Хвора на туберкульоз літня людина (75 років) споживає біля 70г білка на добу, а екскреція загального азоту складає 25г на добу. Про що може свідчити такий стан азотистого обміну:

A. * Посилення процесів розпаду білків

B. Розпад та синтез білків урівноважений

C. Посилення швидкості синтезу білків

D. Зниження швидкості синтезу ліпідів

E. Посилення швидкості синтезу амінокислот

61. У пацієнта з вираженими набряками виявили зміну білкового складу плазми крові. Порушення в складі якої фракції білків особливо сильно впливає на колоїдно-осмотичний тиск?

A. * Альбумінів

B. α - глобулінів

C. β - глобулінів

D. γ - глобулінів

E. Фібриногену

62. У одного з новонароджених близнят з масою тіла 2,4 кг, через 5 годин після народження з”явився тремор, ціаноз, збудження, судоми. Рівень глюкози в крові 1,5 ммоль/л. Для якого стану характерний даний симптомокомплекс?

А. *Неонатальна гіпоглікемія

B. Фізіологічна гіпоглікемія

C. Гіпоглікемічна кома

D. Гіперглікемія

E. Лактоацидотична кома

63. Хлопчик 11 років, хворий на цукровий діабет, поступив до лікарні в стані несвідомості, лежить без рухів, з закритими очима, будь-яка реакція на зовнішні подразники відсутня. Вміст глюкози в крові складає 2,0 ммоль/л. Що може бути причиною коми у хворого?

A. *Абсолютний надлишок інсуліну

B. Абсолютна нестача інсуліну

C. Інфекція

D. Стрес

E. Охолодження

64. За результатами тесту з навантаженням глюкозою толерантність до вуглеводів у пацієнта не порушена. При цьому в сечі хворого визначається глюкоза [5 ммоль/л]. Встановлено діагноз нирковий діабет. Що є головним механізмом глюкозурії в даному випадку?

A. *Зменшення активності ферментів реабсорбції глюкози в нирках

B. Збільшення активності ферментів реабсорбції глюкози в нирках

C. Перевищення порогу реабсорбції для глюкози

D. Посилення секреції глюкози в нирках

E. Збільшення фільтрації глюкози в сечу

65. У ракового хворого розвинулися набряки. Що відіграє провідну роль в розвитку набряку?

A. * Гіпопротеїнемія

B. Гіперпротеїнемія

C. Збільшення онкотичного тиску тканин

D. Зменшення онкотичного тиску тканин

E. Диспротеїнемія

66. У пацієнта через довготривалу блювоту відбулася значна втрата шлункового соку, що стало причиною порушення КЛР в організмі. Яка з перерахованих форм порушення КЛР має місце?

A. * Негазовий алкалоз

B. Газовий ацидоз

C. Негазовий ацидоз

D. Газовий алкалоз

E. Метаболічний ацидоз

67. У хворого, в результаті харчовго отруєння, розвинулася невгамовна блювота. рН крові 7,5, рСО2 артеріальної крові 50мм.рт.ст., буферні основи – 38 ммоль/л, зсув буферних основ +6 ммоль/л. Яке порушення кислотно-лужного стану має місце?

A. * Негазовий алкалоз

B. Газовий алкалоз

C. Негазовий ацидоз

D. Газовий ацидоз

E. Нирковий ацидоз

68. У пацієнта після важкої травми розвинулася нирково-печінкова недостатність. Залишковий азот – 60 ммоль/л, сечовина – 8 ммоль/л, аміак – 300 мкм/л, креатинін – 380 мкм/л. Яке порушення азотистого балансу?

A. * Змішана азотемія

B. Ниркова азотемія

C. Печінкова азотемія

D. Позитивний азотистий баланс

E. Негативний азотистий баланс

69. У жінки 45 років відсутні симптоми діабету, але натще визначається підвищений вміст глюкози в крові (7,2 ммоль/л). Який наступний тест необхідно провести?

A. *Визначення толерантності до глюкози

B. Визначення залишкового азоту в крові

C. Визначення глюкози крові

D. Визначення глюкози у сечі

E. Визначення гліколізованого гемоглобіну

70. Чоловік 58 років страждає на атеросклероз судин головного мозку. При обстеженні виявлена гіперліпідемія. Вміст якого класу ліпопротеїнів найбільш вірогідно буде підвищений при дослідженні сироватки крові?

A. *Ліпопротеїни низької щільності

B. Ліпопротеїни високої щільності

C. Комплекси жирних кислот з альбумінами

D. Хіломікрони

E. Ліпопротеїни проміжної щільності

71. Під час профогляду жінка 38 років пред'явила скарги на постійне відчуття спраги, сухість в роті. Лабораторні дослідження показали: вміст глюкози в крові 10,7 ммоль/л, відмічається глюкозурія, концентрація в крові 17- ОКС не змінена. Для якого стану найбільш характерні ці зміни?

А. *Цукрового діабету

B. Нецукрового діабету

C. Ниркового діабету

D. Стероїдного діабету

E. Аліментарної глюкозурії

72. У хворого Е. знайдена хвороба Барракера-Сімона (прогресивна ліподистрофія): спостерігається зникнення жиру з жирової тканини на голові і грудній клітці з одночасним відкладенням його в нижній половині тіла. У патогенезі цього захворювання провідну роль виконує:

А. *Пошкодження центрів проміжного спинного мозку і вузлів симпатичного стовбура

B. Порушення функції печінки

С. Гіпокортицизм

D. Гіпофункція підшлункової залози

E. Гіпофункція передньої частки гіпофіза

73. У чоловіка 35 років, який протягом останніх 10 років дотримується вегетаріанської дієти, погіршало самочуття. При обстеженні виявлено негативний азотистий баланс. Як називається такий вид голодування?

А.*Якісне

B. Повне

С. Фізіологічне

D. Неповне

E. Кількісне

74. У хворого внаслідок емболії легеневої артерії розвинувся респіраторний алкалоз. Що є безпосередньою причиною цього?

A. * Гіпервентиляція

B. Гіповентиляція

C. Гіперкапнія

D. Артеріальна гіпоксемія

E. Брадипное

75. Пацієнт, який хворіє на цукровий діабет, доставлений в лікарню без свідомості, артеріальний тиск знижений, дихання типу Куссмауля, з рота чути запах ацетону. Провідним механізмом для розвитку цієї коми є:

A. *Накопичення в крові кетонових тіл

B. Накопичення іонів калію

C. Накопичення іонів натрію

D. Накопичення іонів хлору

E. Накопичення сечовини

76. Під час експерименту тварині протягом тривалого часу вводили токсичні речовини, після чого з”явилися симптоми гіперглікемії, поліурії, постійної спраги, надмірного апетиту, які зникали на деякий час після введення гормону інсуліну. Ураження якого органу викликали токсичні речовини?

A.*Підшлункової залози

B. Прищитовидних залоз

C. Наднирників

D. Щитовидної залози

E. Гіпоталамусу

77. У хворого віком 28 років спостерігалося надмірне потовиділення, тахікардія, сильний голод та збудження. Відомо, що хворий отримує інсулін з лікувальною метою. Зменшення концентрації глюкози нижче якого рівня потрібно для появи таких симптомів?

A. * 2.78 ммоль/л

B. 1,78 ммоль/л

C. 3.78 ммоль/л

D. 4.78 ммоль/л

E. 0.78 ммоль/л

78. В другій половині вагітності у жінки відмічалася гіпертензія, протеїнурія, гіпопротеїнемія, з'явилися набряки. Головна роль в розвитку набряків в даному випадку належить:

A. *Зниженню онкотичного тиску крові

B. Альдостеронемії

C. Венозній гіперемії

D. Лімфостазу

E. Запаленню

79. Хвора К., скаржиться на зниження температури тіла, набряки, м'язову слабкість, діарею. З анамнезу відомо, що мала місце довготривала дієта. При обстеженні виявлено: гіпопротеїнемія, зменшення основного обміну, атрофічний синдром. Назвіть вид голодування?

A. *Білкове

B. Жирове

C. Вуглеводне

D. Мінеральне

E. Водне

80. В приймальне відділення лікарні доставлений хворий М.,50 років, який страждає на цукровий діабет. Яке з нижчеперерахованих визначень характеризує стан гіперглікемії?

A. *Підвищення рівня глюкози в крові

B. Зниження рівня глюкози в крові

C. Виведення глюкози з сечею

D. Збільшення вмісту кетонових тіл у крові

E. Виведення кетонових тіл з сечею

81. В крові пацієнта вміст глюкози натще був 5,7 ммоль/л, через 1 годину після цукрового навантаження становив 11,2 ммоль/л, через 2 години 5,9 ммоль/л. Такі показники характерні для:

A. *Хворого на прихований цукровий діабет

B. Здорової людини

C. Хворого на інсулінонезалежний цукровий діабет

D. Хворого на інсулінозалежний цукровий діабет

E. Хворого з синдромом Іценко-Кушінга

82. Жінка 40 років потрапила до лікарні з діагнозом харчове отруєння. У хворої спостерігається багаторазове блювання, підвищена температура. Зменшення яких мінеральних речовин в крові відбувається в цьому випадку?

A. *Хлориду натрію

B. Гідрокарбонату натрію

C. Хлориду калію

D. Гідрокарбонату калію

E. Цитрату кальцію

83. У хворого з цукровим діабетом виявлена гіперглікемія 19 ммоль/л, яка клінічно проявляється глюкозурією, поліурією, полідипсією. Який з представлених механізмів відповідальний за розвиток глюкозурії?

А. *Порогова реабсорбція глюкози

B. Неферментативне глікозування білків

C. Поліурія

D. Полідипсія

E. Дегідратація тканин

84. У дитини 7 років встановлено діагноз – гіперпластичне ожиріння. Що могло стати його причиною?

A. *Перегодовування в грудному віці

B. Первинна недостатність лептину

C. Вторинна недостатність лептину

D. Гіперпродукція інсуліну

E. Психогенна булемія

85. У хворого 40 років встановлено діагноз – вторинне ожиріння. Що могло стати його причиною?

A .*Енцефаліт

B. Гіпопродукція кортизолу

C. Спадкове порушення жирового обміну

D. Зменшення утворення інсуліну

E. Перегодовування в грудному віці

86. У хворого встановлено діагноз – вторинне ожиріння. Що могло стати його причиною?

A.* Гіперпродукція кортизолу

B. Гіпопродукція кортизолу

C. Спадкова гіперхолестеринемія

D. Зменшення утворення інсуліну

E. Перегодовування у грудному віці

87. Людина в екстремальних умовах тривалий час голодувала. Які симптоми порушення гомеостазу виникали в першу чергу?

A *Поява набряків внаслідок гіпопротеїнемії

B Прояви легеневої недостатності

C Порушення сечоутворюючої функції нирок

D Прояви серцевої недостатності

E Гіповолемія

88. У немовля с пілороспазмом внаслідок блювання, що часто повторювалося, з'явилися слабкість, гіподинамія, іноді судоми. Яка форма порушення кислотно-основного стану в нього спостерігається?

A. *Видільний алкалоз

B. Екзогенний негазовий ацидоз

C. Газовий алкалоз

D. Метаболічний ацидоз

E. Видільний ацидоз

89. Причиною захворювання на пелагру може бути переважне харчування кукурудзою і зниження у раціоні продуктів тваринного походження. Відсутність у раціоні якої амінокислоти призводить до даної патології?

A. Метіонін

B. Ізолейцин

C. Фенілаланін

D. Гістидин

E. *Триптофан

90. У фізично здорових молодих курсантів після важкого фізичного навантаження при одноденному пішому переході на 50 км в сечі виявлено білок, рівень якого в середньому не перевищував 1 г/л. Який різновид протеїнурії мав місце?

A. Аліментарна

B. *Маршова

C. Несправжня

D. Дегідратаційна

E. Органічна

91. До лікаря звернулась мати з приводу поганого самопочуття дитини - відсутність апетиту, поганий сон, дратівливість. При біохімічному дослідженні в крові виявлено відсутність ферменту глюкоцереброзидази. Для якої патології це характерно?

A. *Хвороба Тея-Сакса

B. Хвороба Гірке

C. Хвороба Гоше

D. Хвороба Німана-Піка

E. Хвороба Помпе

Гарячка

1. У|в,біля| хворого на крупозну| пневмонію має місце гарячка| з температурою тіла 39°С |із|, при цьому| добові коливання| температури| не перевищували| 1'С|із| впродовж| 9 діб. До якого| типу температурних| кривих| відноситься ця| гарячка|?

A. *Постійна

B. Гектична

C. Ремітуюча

D. Гіперпіретична

E. Поворотна

2. При запальних| процесах| в організмі починається синтез білків “гострої фази|”. Які речовини| є стимуляторами їх синтезу?

A. *Інтерлейкін-1

B. Імуноглобуліни

C. Інтерферони

D. Біогенні аміни

E. Ангіотензини

3. Для розвитку| гарячкових| станів характерним| є зростання| рівня білків “гострої фази| “ – церулоплазміну, фібриногену, С-реактивного протеїну. Вкажіть можливий| механізм цього| зростання?

A. *Стимулюючий вплив| ІЛ-1 на гепатоцити|

B. Руйнівна дія підвищеної температури| на клітини організму

C. Проліферативна дія ІЛ-2 на Т-лімфоцити

D. Проліферативна дія ІЛ-2 на В-лімфоцити

E. Дегрануляція тканинних| базофілів

4. У|в,біля| пацієнта після переливання| крові в кількості 200 мл| піднялась температура до 37,9°С. Яка з нижче| названих| речовин| найбільш ймовірно призвела| до підвищення температури| тіла?

A. *Інтерлейкін 1

B. Інтерлейкін 2

C. Фактор|фактор| некрозу пухлин|

D. Інтерлейкін 3

E. Інтерлейкін 4

5. У|в,біля| хворого розвинулася| гарячка|, яка супроводжувалася| зміщенням установчої точки|точка| терморегуляційного центру на більш високий| рівень, з послідовним чергуванням| наступних| стадій: Incrementi, fastigii, decrementi. При якому| захворюванні можуть| спостерігатися подібні зміни?

A. *Гостра пневмонія

B. Акромегалія

C. Цукровий діабет

D. Ренальний діабет

E. Гіпертрофія міокарда

6. При обстеженні хворого виявлені наступні клінічні прояви|вияви|: шкірні покриви рожеві, теплі на дотик|, сухі, ЧСС| - 92/хв.,ЧД - 22/хв., температура тіла - 39,2°C. Яке співвідношення процесів утворення| і віддачі тепла в описаному| періоді?

A.* Теплопродукція дорівнює тепловіддачі

B. Теплопродукція перевищує тепловіддачу

C. Теплопродукція нижче| за тепловіддачу

D. Зниження тепловіддачі на фоні незміненої теплопродукції

E. Посилення теплопродукції без зміни тепловіддачі

7. Після|потім| введення|вступ| пірогена| у|в,біля| хворого А. підвищилася температура, шкірні покриви стали блідими, холодними на дотик, з'явився|появився| озноб, збільшилося споживання|вжиток| кисню. Як змінюються процеси терморегуляції в описаному періоді

A. *Знижується тепловіддача

B. Збільшується теплопродукція|

C. Тепловіддача рівна теплопродукції|

D. Знижується теплопродукція|

E. Збільшується теплопродукція|

8. При дії на організм низької температури| підвищується активність механізмів терморегуляції. Які з нижчевказаних| механізмів найбільш ефективно| обмежують| тепловіддачу?

A.*Спазм судин| шкіри

B. Брадипное

C. Брадикардія

D. Розширення судин| шкіри

E. Посилення потовиділення

9. Чоловік 25 років скаржиться| на загальну| слабкість, озноб, біль у|в,біля| горлі. Об'єктивно: почервоніння в області мигдаликів. Температура тіла 38.6°С|. Які з перелічених клітин є головним| джерелом| ендогенних| пірогенів, що викликають| гарячку| у хворого?|в,біля|

A. *Нейтрофіли

B. Еозинофіли

C. Лімфоцити

D. Базофіли

E. Тучні клітини

10. У|в,біля| хворого з пневмонією виникла| гарячка|. Що безпосередньо| спричинює зміну установочної точки температури| в нейронах гіпоталамуса цього| хворого?

A. * Простогландини Е1, Е2

B. Ендотоксин

C. Екзотоксин

D. Інтерлейкін-2

E. Тромбоцитарний фактор||зріст|

11. У хворого з гарячкою|морозити|, спостерігається блідість|побліднути| шкірних покривів, "гусяча шкіра", озноб, тахікардія. Якій стадії гарячки|пропасниця| відповідає цей стан?

A.* Стадії підйому температури

B. Стадії стояння температури

C. Стадії зниження температури

D. Стадії ремісії

E. -

12. Жінка 23 років поступила|вчинити| в стаціонар з діагнозом гостра| пневмонія. Захворіла гостро|, 2 дні назад, коли з'явився| озноб з підвищенням температури| тіла до 39, слабкість, сухий| кашель. Який з перерахованих| медіаторів запалення| має властивості ендогенного| пірогену?

A.* Інтерлейкін 1

B. Тромбоксан| А2

C. Гістамін

D. Серотонін

E. Брадикінін

13. У|в,біля| хворого вдень раптово піднялася|підійнялася| температура до 39,5°С і через 6 годин повернулася до норми. На другу добу напад|приступ| повторився і температура досягла 41,5°С, період апірексії наступив|настати| через 8 годин. Який тип температурної кривої?

A.* Переміжний

B. Послабляючий|

C. Септичний

D. Виснажливий

E. Постійний

14. У|в,біля| робітника, який| працював| влітку в|в,біля| щільному костюмі, різко підвищилася температура тіла, з’явилися| задишка|, тахікардія, нудота|, судоми|, втрата| свідомості. Що стало причиною важкого стану робітника?

A. * Зниження тепловіддачі

B. Підвищення теплопродукції

C. Підвищення тепловіддачі

D. Зниження теплопродукції

E. Тепловіддача дорівнює теплопродукції

15. У|в,біля| хворого з|із| інфекційним захворюванням температура тіла через добу підвищувалась до 39,5-40,5°С і трималася на цій висоті близько години, а потім поверталася до початкового|вихідний| рівня. Який тип гарячкової кривої описаний в даному випадку?